Download as pdf or txt
Download as pdf or txt
You are on page 1of 100

AP Comparative

Government and Politics

Practice Exam #3
and Notes
For the
Spring 2020
Exam
© 2020 College Board. College Board, Advanced Placement, AP,
AP, AP
AP Central,
Central, and
and the acorn logo are registered
registered
trademarks of College Board.
Visit College Board on the web: collegeboard.org.

2 AP Comparative Government & Politics Practice Exam


Contents

I. Practice Exam
Exam Content and Format. . . . . . . . . . . . . . . . . . . . . . . . . . . . . . . . . . . . . . . . . . . . . . . . . 4
Administering the Practice Exam. . . . . . . . . . . . . . . . . . . . . . . . . . . . . . . . . . . . . . . . . . 4
Answer Sheet for Multiple-Choice Section. . . . . . . . . . . . . . . . . . . . . . . . . . . . . . . . . 6
AP® Comparative Government and Politics Practice Exam. . . . . . . . . . . . . . . . 7

II. Notes on the Practice Exam


Multiple-Choice Section. . . . . . . . . . . . . . . . . . . . . . . . . . . . . . . . . . . . . . . . . . . . . . . . . . 44
Answer Key and Question Alignment to Course Framework . . . . . . . . . . . . 87
Free-Response Section . . . . . . . . . . . . . . . . . . . . . . . . . . . . . . . . . . . . . . . . . . . . . . . . . . . . 89

Contact Us. . . . . . . . . . . . . . . . . . . . . . . . . . . . . . . . . . . . . . . . . . . . . . . . . . . . . . . . . . . . . . . . . . . . . . . . . 99

AP Comparative Government & Politics Practice Exam 3


Practice Exam

Exam Content and Format


The AP Comparative Government and Politics Exam is 2 hours and 30 minutes
long. There are two sections:
• Section I is 1 hour and consists of 55 multiple-choice questions, accounting
for 50 percent of the final score.

• Section II is 1 hour, 30 minutes and consists of 4 free-response questions


accounting for 50 percent of the final score.

Administering the Practice Exam


This section contains instructions for administering the AP Comparative
Government and Politics Practice Exam. You may wish to use these instructions
to create an exam situation that resembles an actual administration. If so, read
the indented, boldface directions to the students; all other instructions are for
administering the exam and need not be read aloud. Before beginning testing,
have all exam materials ready for distribution. These include test booklets and
answer sheets. (Reminder: Final instructions for every AP Exam are published in
the AP Exam Instructions book.)

SECTION I: Multiple Choice

When you are ready to begin Section I, say:

Section I is the multiple-choice portion of the exam. Mark all of your


responses on your answer sheet, one response per question. If you
need to erase, do so carefully and completely. Your score on the
multiple-choice section will be based solely on the number of questions
answered correctly.

You have 1 hour for this part. Open your Section I booklet and begin.

Note Start Time ________. Note Stop Time ________. After 50 minutes, say:

There are 10 minutes remaining.

After 10 minutes, say:

Stop working. I will now collect your Section I booklet and


multiple-choice answer sheet.

There is a 10-minute break between Sections I and II.

4 AP Comparative Government & Politics Practice Exam


SECTION II: Free Response

After the break, say:

Section II is the free-response portion of the exam.

You have 1 hour and 30 minutes to complete Section II. You may use
any blank space of the page the questions or documents are printed on
to organize your answers and for scratch work, but you must write your
answers on the lined pages provided for the free-response questions.

The suggested writing time for Question 1 is 10 minutes. After


10 minutes, you will be advised to go on to the next question. The
suggested writing time for questions 2 and 3 is 20 minutes each. After
20 minutes, you will be advised to go on to the next question. The
suggested writing time for Question 4 is 40 minutes. Open your
Section II booklet and begin.

Note Start Time ________. Note Stop Time ________. After 10 minutes, say:
It is recommended that you now move on to the next question.

Note Start Time ________. Note Stop Time ________. After 20 minutes, say:
It is recommended that you now move on to the next question.

Note Start Time ________. Note Stop Time ________. After 20 minutes, say:
It is recommended that you now move on to the next question.

After 30 minutes, say:


There are 10 minutes remaining.

After 10 minutes, say:


Stop working and close your exam booklet. Put your exam booklet on
your desk, face up. Remain in your seat, without talking, while the exam
materials are collected.

Collect a Section II booklet from each student and check that each student wrote
his or her answers on the lined pages corresponding to each question. Then say:
The exam is over. You are now dismissed.

AP Comparative Government & Politics Practice Exam 5


Name:

AP® Comparative Government and Politics


Answer Sheet
for Multiple-Choice Section

No. Answer No. Answer


1 29
2 30
3 31
4 32
5 33
6 34
7 35
8 36
9 37
10 38
11 39
12 40
13 41
14 42
15 43
16 44
17 45
18 46
19 47
20 48
21 49
22 50
23 51
24 52
25 53
26 54
27 55
28

6 AP Comparative Government & Politics Practice Exam


AP Comparative Government and
®

Politics Exam
SECTION I: Multiple Choice

DO NOT OPEN THIS BOOKLET UNTIL YOU ARE TOLD TO DO SO.

Instructions
At a Glance
Section I of this exam contains 55 multiple-choice questions.
Total Time
1 hour Indicate all of your answers to the multiple-choice questions on the answer sheet. No credit
Number of Questions will be given for anything written in this exam booklet, but you may use the booklet for
55 notes or scratch work.
Percent of Total Score
50% Use your time effectively, working as quickly as you can without losing accuracy. Do not
Writing Instrument spend too much time on any one question. Go on to other questions and come back to
Pencil required the ones you have not answered if you have time. It is not expected that everyone will
know the answers to all of the multiple-choice questions.
Your total score on the multiple-choice section is based only on the number of questions
answered correctly. Points are not deducted for incorrect answers or unanswered
questions.

AP Comparative Government & Politics Practice Exam 7


The inclusion of source material in this exam is not intended as an
endorsement by the College Board or ETS of the content, ideas, or
values expressed in the material. The material has been selected by
the government and politics faculty who serve on the AP
Comparative Government and Politics Development Committee. In
their judgment, the material printed here reflects various aspects of
the course of study on which this exam is based and is therefore
appropriate to use to measure the skills and knowledge of this
course.

8 AP Comparative Government & Politics Practice Exam


COMPARATIVE GOVERNMENT AND POLITICS
SECTION I
Time—1 hour
55 Questions
Directions: Each of the questions or incomplete statements below is followed by four suggested answers or
completions. Select the one that is best in each case and then enter the letter in the corresponding space on the
answer sheet.

1. Which of the following best explains a 3. Which of the following best describes pluralist
consequence of the way power is distributed in a systems?
unitary system?
(A) They support economic policies that are
(A) Because power is diffused among several developed and run by the state.
levels of government, responses to the (B) They allow the state to maintain control
needs of citizens are slower and often over citizen input and participation.
contradictory.
(C) They promote competition among
(B) Because power is shared equally between a autonomous groups not linked to the
national government and subnational state.
governments, public policy is static and
(D) They use single peak associations to
cannot change over time.
represent the business sector.
(C) Because power is concentrated in the
national government with some
distribution to subnational levels, policy
making is often efficient.
(D) Because power is centralized in one place, 4. Which is the most common selection method for
the national government makes all members of the United Kingdom’s House of
decisions and there are no other levels of Lords?
governments.
(A) Being nominated and appointed through
political party lists
(B) Being elected by members of the House of
Commons
2. Which of the following data sources best (C) Winning a seat through a national election
measures economic inequality?
(D) Being nominated by the prime minister and
(A) Gini index approved by the monarch
(B) Human Development Index (HDI)
(C) Gross domestic product (GDP) per capita
(D) Freedom House civil liberties score

GO ON TO THE NEXT PAGE.

AP Comparative Government & Politics Practice Exam 9


5. Which of the following best explains why 7. Which of the following best explains why both
countries rich in oil are susceptible to economic Nigeria and the United Kingdom have two major
crises? political parties?

(A) Undervaluation of their currencies makes it (A) Both countries’ politics focus on economic
hard for them to import materials needed issues, which leads to one party
for investment. representing business interests while the
(B) Inequality between rich and poor makes other party represents labor interests.
them susceptible to social conflict. (B) Both countries’ electoral systems use
(C) Lack of economic diversification makes single-member districts with
them vulnerable to debt when oil prices first-past-the-post rules, which tend to
fall. favor catch-all parties.
(D) High taxation of their citizens makes them (C) Nigeria’s main cleavage is ethnic, while the
susceptible to budgetary shortfalls when United Kingdom’s main cleavage is
oil prices fall. regional, favoring two parties that
represent those ethnic and regional
cleavages.
(D) Nigeria’s corrupt electoral authority
suppresses small parties, while the United
6. Which of the following pairs of countries both Kingdom’s use of proportional
require that the chief executive receive an representation underrepresents small
absolute majority of the public’s votes to be parties.
elected?

(A) Iran and Russia


(B) Iran and Mexico
(C) Nigeria and the United Kingdom 8. Which of the following best explains a
demographic consequence of economic policies
(D) Mexico and Nigeria in Mexico?

(A) Economic liberalization policies influenced


job creation in northern Mexico,
prompting migration from the south to the
north.
(B) Economic liberalization policies failed in
northern Mexico, prompting migration to
southern Mexico.
(C) Economic liberalization policies influenced
job creation in rural areas of Mexico,
prompting the growth of major cities
throughout the country.
(D) Economic liberalization policies were
successful in southern Mexico, prompting
population growth there.

GO ON TO THE NEXT PAGE.

10 AP Comparative Government & Politics Practice Exam


9. Which of the following is the purpose of the 11. Which of the following is an accurate
Great Firewall in China? description of the Mexican judiciary?

(A) To expand infrastructure to rural areas in (A) There is no functioning Supreme Court in
the west Mexico.
(B) To facilitate migration from rural to urban (B) Mexico’s judiciary does not have the
areas power of judicial review.
(C) To increase global influence through (C) Supreme Court justices are appointed by
cross-border road projects the president and confirmed by the
(D) To limit political criticism on social media Senate.
(D) Supreme Court justices are appointed to a
life term.

10. Which of the following is the best explanation of


how two AP Comparative Government and
Politics course countries have responded to 12. An interest group system that controls access to
global market forces? policy making by relying on state-sanctioned
groups to represent labor, business, and
(A) Mexico has withdrawn from the North agricultural sectors is
American Free Trade Agreement
(NAFTA) to protect domestic industry, (A) socialist
while China has privatized its economy to (B) corporatist
promote competition.
(C) populist
(B) The United Kingdom has nationalized the
(D) pluralist
banking industry to regulate credit
markets, while Iran has allowed the
Revolutionary Guard to supervise the
economy to protect national interests.
(C) China has created special economic zones
(SEZs) to increase foreign direct
investment, while Mexico has privatized
some of its oil industry to increase
competition.
(D) The United Kingdom has nationalized
natural resources to increase state
revenue, while Russia has privatized
natural resources to increase competition.

GO ON TO THE NEXT PAGE.

AP Comparative Government & Politics Practice Exam 11


Questions 13 - 15 refer to the line graph.

LIFE EXPECTANCY AT BIRTH IN THE SOVIET UNION AND THE RUSSIAN FEDERATION, 1985–2017

Source: The World Bank, 2017.

13. Which of the following accurately describes a 14. Which of the following describes the most likely
trend shown in the chart? political result of the change in life expectancy
represented in the chart from 2005 to 2017 ?
(A) Life expectancy steadily rose in the Soviet
Union under the leadership of Mikhail (A) An increase in international humanitarian
Gorbachev (1985–1991). aid if foreign donors perceive more
(B) Life expectancy was consistently lower assistance is needed
under the Soviet Union (1922–1991) than (B) An increase in legitimacy if citizens
it was under the Russian Federation from believe the change is a result of effective
1991 to 2017. government policies
(C) Life expectancy consistently rose once (C) A decrease in tax revenues if the average
Vladimir Putin became president of the age of retirement increases
Soviet Union in 2000. (D) A decrease in voter turnout as elderly
(D) When Vladimir Putin became president in citizens are not likely to vote
2000, life expectancy was declining.

GO ON TO THE NEXT PAGE.

12 AP Comparative Government & Politics Practice Exam


15. Using your knowledge and the data in the chart, 17. Which of the following is an accurate
which of the following is the most significant comparison of a head of state and a head of
limitation? government?

(A) The data are based on normative (A) A head of government generally serves as
conclusions derived from census data, so the leader in charge of conducting foreign
they cannot be interpreted in a policy, while a head of state serves as the
meaningful way. leader with sole responsibility for
(B) Because the Soviet Union dissolved in executing domestic policy.
1991, the data are not useful in any (B) A head of state serves as the chief
analyses of the Soviet Union or Russia executive responsible for enforcing laws
over time. and treaties, while a head of government
(C) The data do not distinguish between male serves as the leader of the legislature with
and female life expectancy, which is an sole responsibility for creating policy.
important distinction in Russia. (C) A head of government generally serves as
(D) There are not enough data points to detect the symbolic political representative,
a clear trend regarding life expectancy at while a head of state is the official
birth in Russia after 1991. responsible for running the government.
(D) A head of state generally serves as the
symbolic political representative of the
nation, while a head of government is the
official responsible for running the
16. Which of the following institutions most limit government.
the exercise of independent legislative power?

(A) Mexico’s National Electoral Institute (INE)


and Nigeria’s Independent National
Electoral Commission (INEC) 18. Which of the following is an accurate
(B) China’s Standing Committee of the explanation of an advantage of executive term
Politburo and Iran’s Guardian Council limits?
(C) United Kingdom’s House of Lords and
(A) Executive term limits allow legislators to
Nigeria’s president
develop coherent policies over time.
(D) Iran’s Supreme Leader and the United
(B) Executive term limits allow executives
Kingdom’s monarchy
greater control over the legislative
process.
(C) Executive term limits can prevent
executives from consolidating power.
(D) Executive term limits can weaken
accountability.

GO ON TO THE NEXT PAGE.

AP Comparative Government & Politics Practice Exam 13


19. Which of the following statements is true about 21. Which of the following accurately explains a
the effects of cleavages in two course countries? consequence of economic liberalization policies
in China?
(A) Regional cleavages in both Iran and the
United Kingdom have resulted in groups (A) Economic liberalization in China created
successfully gaining independence. job growth on the east coast, causing
(B) Religious cleavages have led to significant migration from the west to the east.
separatist movements in both Nigeria and (B) Economic liberalization in China created
Mexico, resulting in a military response job growth in urban areas, driving
from the state. migration from the east to the west.
(C) Regional cleavages have resulted in (C) Economic liberalization in China created
demands for greater autonomy and job growth in rural areas, driving
independence in the United Kingdom, migration from the cities to the
whereas in Mexico, regional cleavages countryside.
have only resulted in demands for greater (D) Economic liberalization in China created
autonomy. job growth in the north, driving migration
(D) Ethnic cleavages have not resulted in from the south to the north.
conflict in China and Russia because the
state has given fair representation in
government to these groups.
22. Which of the following is an accurate
comparison of the current political party systems
in Russia and China?
20. Which of the following is an accurate
comparison of sources of legitimacy in Iran and (A) China has a two-party system, while
Mexico? Russia has a dominant-party system.
(B) China has a one-party system, while Russia
(A) In Iran, the regime primarily derives its has a dominant-party system.
legitimacy from the Assembly of Experts,
(C) China has a multiparty system, while
while in Mexico, the primary source of
Russia has a one-party system.
legitimacy is the rule of law.
(D) China has a dominant-party system, while
(B) In Iran, the regime primarily derives its
Russia has a one-party system.
legitimacy from the rule of law, while in
Mexico, the primary source of legitimacy
is the Supreme Court.
(C) In Iran, the regime primarily derives its
legitimacy from the Supreme Leader,
while in Mexico, the primary source of
legitimacy is the president.
(D) In Iran, the regime primarily derives its
legitimacy from religion, while in
Mexico, the primary source of legitimacy
is the constitution.

GO ON TO THE NEXT PAGE.

14 AP Comparative Government & Politics Practice Exam


Questions 23 - 25 refer to the passage.

The 2009 presidential election in the Islamic Republic of Iran was one of the most transformative events in Iran’s
modern history. It bared important schisms within the .·.·. political system and pitted two key camps against one
another, each with a very different vision of what Iran should be and what it should become. It appeared to solidify
. . . under President Mahmoud Ahmadinejad and continued an ongoing militarization of Iranian politics led by the
Islamic Revolutionary Guard Corps. . . . The widespread fraud alleged by the leading opposition candidate, former
Prime Minister Mir Hossein Mousavi, and the .·.·. (political system’s) repressive response irrevocably shattered an
unspoken contract between the government and the people—one in which the theocratic government had allowed
some popular political participation and limited personal space in return for the people’s acquiescence to the status
quo.

Even the highest authority in Iran, the Supreme Leader, Ayatollah Ali Khamenei, did not escape censure by the
opposition—a traditional “red line” in Iranian politics that clerics, politicians, and voters alike crossed numerous
times after the polls closed. Previously, Khamenei had portrayed himself as above the often-brutal factional “fray” in
Iran. But now he came down decisively on the side of Ahmadinejad and his hard-line allies and used the
Revolutionary Guard to preserve the status quo. In so doing, he altered the role of the office he occupied.
Source: Alireza Nader, David E. Thaler, and S.R. Bohand. The Next Supreme Leader: Succession in the
Islamic Republic of Iran. Rand National Defense Research Institute, 2011

23. Which of the following best describes the 24. Which of the following is a role of the supreme
authors’ evidence that Supreme Leader leader and explains why Khamenei has the
Khamenei sided with President Ahmadinejad in authority to take the actions described by the
the aftermath of the 2009 presidential election in authors?
Iran?
(A) Chief legislator
(A) Khamenei changed the role of president to (B) Head of government
take a more active role in politics.
(C) Commander in chief
(B) Khamenei used the Revolutionary Guard to
(D) Chief justice of Iran
preserve the status quo by suppressing
protests.
(C) Khamenei altered the results of the election
to favor Ahmadinejad.
(D) Khamenei formed the Green Movement to
counter protests by opponents of the
regime.

GO ON TO THE NEXT PAGE.

AP Comparative Government & Politics Practice Exam 15


25. Which of the following best explains an 27. Which of the following best compares import
implication of the authors’ argument for the substitution policies and neoliberal economic
legitimacy of Iran’s current regime? policies?

(A) The regime’s legitimacy will weaken (A) The goal of import substitution policies is
because of the supreme leader’s change to protect domestic industries, whereas
from his traditionally neutral role in neoliberal policies seek to decrease
politics. reliance on foreign direct investment
(B) The regime’s legitimacy will weaken (FDI).
because of disapproval of the regime’s (B) Import substitution policies are usually
electoral practices by the global adopted by countries with developed
community. economies, whereas neoliberal policies
(C) The regime’s legitimacy will strengthen are adopted by countries with high levels
because of the regime’s repression of of inequality.
dissent against Ahmadinejad. (C) Import substitution policies support
(D) The regime’s legitimacy will strengthen reducing deficits, whereas neoliberal
because of Iranians’ fear that the policies support raising taxes.
Revolutionary Guards will harshly repress (D) Import substitution policies include
any protest. increasing tariffs, whereas neoliberal
policies encourage lower tariffs.

26. Which of the following best explains an


important step in the democratic consolidation 28. Which of the following best describes the
process? difference between fascism and communism?

(A) Removing civil liberties from any group (A) Fascism is the belief in the interests and
that protests against the government helps rights of the common people over that of
to maintain order. the elites, and communism is the belief in
(B) Establishing rule by law means the nationalization of all industries.
government officials are treated like (B) Fascism is the belief in a nationalist
everyone else. ideology favoring authoritarian rule, and
(C) Reducing public access to government communism is the belief in the removal
documents allows for greater efficiency in of private property and government
decision making. control over the economy.
(D) Allowing for competitive and fair elections (C) Fascism is the belief in the reduction of
gives citizens the right to express their income disparities among all people, and
political opinions. communism is the belief in individual
civil liberties and freedoms over
government intrusion or restriction.
(D) Fascism is the belief in limited government
intervention in the economy and society,
and communism is the belief in the
deregulation of trade and the elimination
of subsidies.

GO ON TO THE NEXT PAGE.

16 AP Comparative Government & Politics Practice Exam


29. Which of the following explains the effect the 31. Rule of law is stronger in the United Kingdom
Guardian Council’s powers have on presidential than it is in Russia because
candidates in Iran?
(A) common law in the United Kingdom
(A) The Guardian Council only allows the two establishes judicial review, whereas the
major political parties to run presidential Constitution in Russia does not
candidates thus limiting the ability of a (B) the United Kingdom is a parliamentary
minor party candidate being elected system, whereas Russia is a
president. semi-presidential system
(B) The Guardian Council excludes nonclerics (C) the United Kingdom has a Supreme Court,
limiting the number of candidates that are whereas Russia does not
eligible to run for president.
(D) the judiciary in the United Kingdom is
(C) The Guardian Council appoints the independent, whereas the judiciary in
president making it important that Russia is not
presidential candidates are loyal to the
Guardian Council.
(D) The Guardian Council restricts the number
of reform-minded presidential candidates
limiting the number of candidates. 32. Which of the following is an accurate
explanation of the difference between political
party systems in Iran and the United Kingdom?

(A) The political party system in Iran reflects


30. Which of the following is a reason why a the principles of Islamic law, while in the
country might implement import substitution United Kingdom, the political party
industrialization (ISI) policies? system is corporatist.
(B) The political party system in Iran has
(A) The country is attempting to attract more evolved to a single-party dominant
private capital and foreign direct system, while in the United Kingdom,
investment. political parties have clear platforms and
(B) The country is attempting to lower tariffs membership rules and compete
and improve its economy. effectively in elections.
(C) The country is attempting to reduce foreign (C) Political parties in Iran are unable to
influence on its economy. effectively support candidates for election
(D) The country is attempting to create more and are not well-established, while in the
state-owned companies. United Kingdom, political parties are free
to compete and the election rules in place
have resulted in a system with two major
parties.
(D) Political parties in Iran have been unable to
raise funds necessary to sustain
themselves, leading to a collapse in the
party system, while in the United
Kingdom, political parties are free to
compete and raise money for elections,
which has resulted in a two-party
dominant system.

GO ON TO THE NEXT PAGE.

AP Comparative Government & Politics Practice Exam 17


33. In which of the following pairs of countries does 34. Which of the following best describes a
the government exercise significant state control consequence of globalization?
of the media?
(A) Multinational corporations reinforce the
(A) Russia and Nigeria sovereignty of governments by bringing
(B) Iran and Mexico them more wealth and power.
(C) China and Russia (B) Globalization usually results in regime
instability caused by international
(D) United Kingdom and Nigeria
pressures for democratization.
(C) Globalization often leads to increased
social tensions, as it tends to challenge
traditional power structures and culture.
(D) Importing cheaper goods from advanced
industrial economies leads to a reduction
in economic inequality.

GO ON TO THE NEXT PAGE.

18 AP Comparative Government & Politics Practice Exam


Questions 35 - 37 refer to the chart.

35. Which of the following statements did most poll 36. Which of the following best explains why many
respondents say described the state of political respondents to the poll agreed that the statement
affairs in Nigeria? “The court system treats everyone fairly”
described Nigeria well?
(A) Elected officials care what ordinary people
think. (A) The Nigerian government has eliminated
(B) No matter who wins an election, things do the Shariah Courts in the north.
not change very much. (B) The Nigerian government has arrested and
(C) The court systems treat everyone fairly. issued lengthy sentences to political
protesters.
(D) Most politicians are corrupt.
(C) The president has been significantly
increasing the size of the Nigerian court
system and has appointed judges who
support the same political ideology.
(D) The Nigerian government has made an
effort to reduce corruption in the judicial
system by protecting the power of judicial
review and independence.

GO ON TO THE NEXT PAGE.

AP Comparative Government & Politics Practice Exam 19


37. Which of the following is an implication of the 40. Which of the following is a consequence of the
chart? bicameral legislature in both Mexico and the
United Kingdom?
(A) The Nigerian independent election
commission will continue to fail to reduce (A) Subnational governments are protected
voter fraud in elections. because the upper chamber must approve
(B) Civil society does not exist in Nigeria to federal intervention in state matters.
challenge corrupt politicians. (B) Civilian control of the military is assured
(C) Nigerian citizens are critical of Nigerian because the upper chamber must approve
politicians and have low levels of political troop deployment.
trust. (C) There is a check on the executive because
(D) The courts are ineffective in addressing the there is fusion of power between the two
needs of Islamic citizens in the north. chambers.
(D) Policy making is slowed because both
chambers participate in the policy-making
process.
38. An illiberal democracy is also known as which
of the following?

(A) A competitive authoritarian regime 41. Which of the following is true of the cabinet in
(B) A substantive democracy both Mexico and Nigeria?
(C) A corporatist democracy
(A) The courts must review all cabinet member
(D) A democratic socialist regime nominations by the executive.
(B) The legislature must approve cabinet
members nominated by the executive.
(C) Cabinet members must be members of the
39. In which two countries does the legislative legislature.
branch have the power of impeachment? (D) The cabinet is solely accountable to the
executive branch.
(A) The United Kingdom and Mexico
(B) China and Russia
(C) Russia and Nigeria
(D) Iran and the United Kingdom

GO ON TO THE NEXT PAGE.

20 AP Comparative Government & Politics Practice Exam


Questions 42 and 43 refer to the passage.

China has taken a multi-pronged approach in its economic relations with Africa, according to Deborah Brautigam
[of] Johns Hopkins School of Advanced and International Studies. China is a significant source of foreign direct
investment in Africa; offers development loans to resource-rich nations . . . invests in agriculture; and develops
special trade and economic cooperation zones in several states . . . “Chinese banks and companies are offering
finance that allows them to secure a greater share of the business deals in Africa as part of their move to ‘go global.’
This brings with it risks for African borrowers—but also opportunities,” writes Brautigam.

Chinese financing comes often in the form of loans and credits provided by the People’s Bank of China, the China
Development Bank, the Export-Import Bank of China, and the China-Africa Development Fund. Between 2000 and
2014, Chinese banks, contractors, and the government loaned more than $86 billion to Africa . . . However, these
large loans are beginning to raise questions about debt loads in African countries, showing indications of a potential
debt crisis.
Council on Foreign Relations, 2019

42. Which of the following best describes the main 43. According to the passage, which of the
point of the passage? following is the most likely implication of
China’s current economic policies toward
(A) China has aggressively pursued a singular African nations?
economic approach in its relations with
Africa. (A) African nations may adopt Communist
(B) Economic relations with Africa have principles to continue to attract Chinese
involved the Chinese government, not aid.
Chinese private sector actors. (B) Indebted African nations may lose
(C) Development loans to African nations sovereignty to the Chinese government
represent a decreasing share of Chinese and large banks.
revenue sources. (C) China may become a member of regional
(D) The scale of borrowing by African African economic organizations such as
countries increases the likelihood of ECOWAS.
future debt problems. (D) China’s economic growth may become
overly dependent on resources from
Africa.

44. Which of the following data sources would be


the most appropriate to use to measure a
country’s protection of civil liberties?

(A) The Gini index


(B) The Human Development Index (HDI)
(C) Gross domestic product (GDP)
(D) Freedom House

GO ON TO THE NEXT PAGE.

AP Comparative Government & Politics Practice Exam 21


45. Which of the following best describes both 47. Which of the following best explains why the
China and Iran? Chinese government allows some political
protests?
(A) Both have a unitary system of government.
(B) Both have a one-party system. (A) To promote democratization among
citizens
(C) Both have a bicameral legislature.
(B) To open political discourse
(D) Both have a semi-presidential system of
government. (C) To spread political information
(D) To act as a release valve for the public

46. Which of the following statements best


compares government regulation of civil society
in Russia with that of the United Kingdom?

(A) Only in Russia must civil society groups


register with the government.
(B) Only in Russia does the government
provide all funding for civil society
groups.
(C) The Russian government monitors civil
society groups much more closely than
does the government of the United
Kingdom.
(D) In both Russia and the United Kingdom the
government frequently jails members of
opposition civil society groups.

GO ON TO THE NEXT PAGE.

22 AP Comparative Government & Politics Practice Exam


Questions 48 and 49 refer to the following map.

Source: World Bank, 2016.

GO ON TO THE NEXT PAGE.

AP Comparative Government & Politics Practice Exam 23


48. Which of the following statements about females 50. Which of the following best explains the impact
with primary school education in 1986 and 2016 of electoral rules on the number of effective
is true? political parties in the legislatures in Russia and
the United Kingdom?
(A) In China, less than 45 percent of females
had a primary school education in 2016. (A) In Russia, electoral rules have resulted in
(B) In Russia, more than 49 percent of females competitive parties and a multiparty
had a primary school education in 2016. legislature, while in the United Kingdom,
electoral rules have allowed regional
(C) In Nigeria, 48 percent of females had a
parties to win a majority of seats in the
primary school education in 1986.
legislature.
(D) In Iran, more than 49 percent of females
(B) In Russia, electoral rules have made it
had a primary school education in 1986.
difficult for parties to compete or gain
representation in the legislature, while in
the United Kingdom, single-member
districts have resulted in a legislature with
49. Which of the following best explains the reason only two parties.
for Iran’s change in percentage of females with (C) In Russia, single-member districts have
primary school education between 1986 and resulted in a two-party dominant system,
2016 ? while in the United Kingdom, electoral
rules limit minor parties and have resulted
(A) After the revolution, the Iranian in a single-party dominant system.
government stopped females from
(D) In Russia, electoral rules have limited the
attending primary school and secondary
number of parties in the legislature, while
school.
in the United Kingdom, electoral rules
(B) Hardliners in the 1990s refused to allow have allowed regional parties to win
females the right to attend school or legislative seats while maintaining a
sporting events. system with two major parties.
(C) The supreme leader changed national laws
to allow women the right to vote and
receive secondary education.
(D) The Iranian government created education 51. Which of the following is a check on the
policies that encouraged females to attend executive in both a presidential and a
primary school. parliamentary system?

(A) There may be imposed time deadlines on


calling new elections.
(B) The lower house of the legislature may
pass an impeachment resolution.
(C) The legislature may refuse to pass
executive proposed legislation.
(D) The Supreme Court may remove the
executive from office.

GO ON TO THE NEXT PAGE.

24 AP Comparative Government & Politics Practice Exam


52. Which of the following best describes an effect 54. The Green Movement in Iran in 2009 was a
of recent demographic change in the United result of which of the following?
Kingdom?
(A) Electoral reforms in the Majles
(A) Recent shortfalls of natural resources have (B) The removal of President Ahmadinejad
resulted in a negative net migration.
(C) Vetting done by the Guardian Council
(B) The government has created special
(D) Corruption in presidential election
economic zones to encourage
immigration.
(C) A positive net migration of immigrants that
has resulted in social and political
tensions. 55. Which of the following statements accurately
(D) There has been an urban to rural describes a cleavage in Iran?
population shift occurring as a result of
(A) The growing Shi’a population in the South
overcrowded cities.
has created a regional cleavage between
the North and South.
(B) The coalition of Azerbaijanis and Kurds is
an ethnic cleavage that threatens the
53. Which of the following best explains how power stability of Iran’s regime.
in the National People’s Congress is (C) A religious cleavage exists between the
constrained? Shi’a Muslims and the minority Iranian
Jews, Christians, and Zoroastrians.
(A) China’s upper house is required to approve
all legislation passed by the National (D) An ethnic cleavage has created tension
People’s Congress concerning the annual between the majority Arab population and
budget. the minority Persian.
(B) The National People’s Congress is directly
elected by the people, so representatives
must take constituents’ needs into
consideration when passing legislation.
(C) The National People’s Congress is
subservient to the Politburo Standing
Committee, which is the actual center of
power in China.
(D) The National People’s Congress is
recognized by the constitution as the most
powerful institution of government and
has no constraints.

GO ON TO THE NEXT PAGE.

AP Comparative Government & Politics Practice Exam 25


END OF SECTION I

IF YOU FINISH BEFORE TIME IS CALLED, YOU MAY


CHECK YOUR WORK ON THIS SECTION.

DO NOT GO ON TO SECTION II UNTIL YOU ARE TOLD TO DO SO.


____________________________________________________________________________

MAKE SURE YOU HAVE DONE THE FOLLOWING:

• PLACED YOUR AP ID LABEL ON YOUR ANSWER SHEET


• WRITTEN AND GRIDDED YOUR AP ID CORRECTLY ON YOUR
ANSWER SHEET
• TAKEN THE AP EXAM LABEL FROM THE FRONT OF THIS BOOKLET AND
PLACED IT ON YOUR ANSWER SHEET

26 AP Comparative Government & Politics Practice Exam


AP Comparative Government and
®

Politics Exam
SECTION II: Free Response

DO NOT OPEN THIS BOOKLET UNTIL YOU ARE TOLD TO DO SO.

At a Glance Instructions
The questions for Section II are printed in this booklet. You may use the pages in this
Total Time
1 hour and 30 minutes
booklet to organize your answers and for scratch work, but you must write your answers
Number of Questions
on the lined pages provided for each question.
4 Write clearly and legibly. Do not skip lines. Begin your response to each question at the top
Percent of Total Score of a new page; completely fill in the circle at the top of each page that corresponds to the
50% question you are answering. Cross out any errors you make; crossed-out work will not be
Writing Instrument scored.
Pen with black or dark
blue ink Manage your time carefully. The proctor will announce the suggested time for each
Questions 1 3 question, but you may proceed freely from one question to the next. You may review your
responses if you finish before the end of the exam is announced.
Suggested Time
10 minutes for
question 1, and
20 minutes each for
questions 2 and 3.

Question 4
Suggested Time
40 minutes
Weight
Question 1: 22%
Question 2: 25%
Question 3: 25%
Question 4: 28%

AP Comparative Government & Politics Practice Exam 27


COMPARATIVE GOVERNMENT AND POLITICS
SECTION II
Time—1 hour and 30 minutes
4 Questions

Directions: You have 1 hour and 30 minutes to answer one conceptual analysis question, one quantitative analysis
question, one comparative analysis question, and one argument essay. Unless the directions indicate otherwise,
respond to all parts of all four questions. It is suggested that you take a few minutes to plan and outline each answer.
It is also suggested that you spend approximately 10 minutes total on question 1, 20 minutes each on questions 2 and
3, and 40 minutes total on question 4. These suggested times do not reflect the weight of the questions as part of
your AP exam score. In your responses, use substantive examples where appropriate. You may plan your answers in
this Questions booklet, but no credit will be given for anything written in this booklet. You will only earn credit for
what you write in the Free Response booklet.
1. a. Describe the Gini index.
b. Describe a difference between the Gini index and the Human Development Index (HDI).
c. Explain how a government policy could affect a state’s Gini index score.
d. Explain how changes in the Gini score reflecting state policies could enhance or diminish an
authoritarian regime’s stability.
__________________________________________________________
Begin your response to this question at the top of a new page in the Free Response booklet
and fill in the appropriate circle indicating the question number.

GO ON TO THE NEXT PAGE.

28 AP Comparative Government & Politics Practice Exam


Source: International IDEA, 2017.

2. A. Using the data in the graph, identify the year with the lowest voter turnout.
B. Using the data in the graph, describe the relationship between total vote and voting-age
population in Mexico between 1993 and 2017.
C. Describe political participation.
D. Using the data in the graph, draw a conclusion about political participation in legislative
elections in Mexico.
E. Explain what the data imply about why democracies encourage political participation.
__________________________________________________________
Begin your response to this question at the top of a new page in the Free Response booklet
and fill in the appropriate circle indicating the question number.

GO ON TO THE NEXT PAGE.

AP Comparative Government & Politics Practice Exam 29


3. Compare how global economic forces influence policies in two different AP Comparative Government and
Politics course countries. In your response, you should do the following.
A. Define economic globalization.
B. Explain how state membership in an international economic organization has influenced economic
policies in two different AP Comparative Government and Politics course countries.
C. Explain why each of the two AP Comparative Government and Politics course countries described in
(B) would choose state membership in an international economic organization.
__________________________________________________________
Begin your response to this question at the top of a new page in the Free Response booklet
and fill in the appropriate circle indicating the question number.

GO ON TO THE NEXT PAGE.

30 AP Comparative Government & Politics Practice Exam


4. Develop an argument that explains whether a two-party system or multiparty systems best enhance the
democratization process.

Use one or more of the following course concepts in your response:

• Accountability
• Legitimacy
• Stability
In your essay, you should do the following:

✓ Respond to the prompt with a defensible claim or thesis that establishes a line of reasoning using one or
more of the provided course concepts.

✓ Support your claim with at least TWO pieces of specific and relevant evidence from one or more course
countries. The evidence should be relevant to one or more of the provided course concepts.

✓ Use reasoning to explain why your evidence supports your claim or thesis, using one or more of the provided
course concepts.
✓ Respond to an opposing or alternate perspective using refutation, concession, or rebuttal.
__________________________________________________________
Begin your response to this question at the top of a new page in the Free Response booklet
and fill in the appropriate circle indicating the question number.

GO ON TO THE NEXT PAGE.

AP Comparative Government & Politics Practice Exam 31


STOP

END OF EXAM
________________________________

MAKE SURE YOU HAVE FILLED IN THE CIRCLE THAT CORRESPONDS TO THE QUESTION YOU
ANSWERED ON EACH PAGE IN THE FREE RESPONSE BOOKLET.

THE FOLLOWING INSTRUCTIONS APPLY TO THE FRONT COVER OF THE SECTION II: FREE
RESPONSE BOOKLET. MAKE SURE YOU HAVE DONE THE FOLLOWING:

• COMPLETED THE IDENTIFICATION INFORMATION AS REQUESTED


• CHECKED THAT YOUR AP EXAM LABEL IS IN THE BOX AT THE TOP
• CHECKED THAT YOUR AP ID LABEL IS IN THE BOX AT THE BOTTOM

32 AP Comparative Government & Politics Practice Exam


Important: Completely fll in the circle Question 1 Question 2 Question 3 Question 4
that corresponds to the question you
are answering on this page.

Begin your response to each question at the top of a new page. Do not skip lines.

Page 2

Use a pen with black or dark blue ink only. Do NOT write your name. Do NOT write outside the box.

AP Comparative Government & Politics Practice Exam 33


Important: Completely fll in the circle Question 1 Question 2 Question 3 Question 4
that corresponds to the question you
are answering on this page.

Begin your response to each question at the top of a new page. Do not skip lines.

Page 3

Use a pen with black or dark blue ink only. Do NOT write your name. Do NOT write outside the box.

34 AP Comparative Government & Politics Practice Exam


Important: Completely fll in the circle Question 1 Question 2 Question 3 Question 4
that corresponds to the question you
are answering on this page.

Begin your response to each question at the top of a new page. Do not skip lines.

Page 4

Use a pen with black or dark blue ink only. Do NOT write your name. Do NOT write outside the box.

AP Comparative Government & Politics Practice Exam 35


Important: Completely fll in the circle Question 1 Question 2 Question 3 Question 4
that corresponds to the question you
are answering on this page.

Begin your response to each question at the top of a new page. Do not skip lines.

Page 5

Use a pen with black or dark blue ink only. Do NOT write your name. Do NOT write outside the box.

36 AP Comparative Government & Politics Practice Exam


Important: Completely fll in the circle Question 1 Question 2 Question 3 Question 4
that corresponds to the question you
are answering on this page.

Begin your response to each question at the top of a new page. Do not skip lines.

Page 6

Use a pen with black or dark blue ink only. Do NOT write your name. Do NOT write outside the box.

AP Comparative Government & Politics Practice Exam 37


Important: Completely fll in the circle Question 1 Question 2 Question 3 Question 4
that corresponds to the question you
are answering on this page.

Begin your response to each question at the top of a new page. Do not skip lines.

Page 7

Use a pen with black or dark blue ink only. Do NOT write your name. Do NOT write outside the box.

38 AP Comparative Government & Politics Practice Exam


Important: Completely fll in the circle Question 1 Question 2 Question 3 Question 4
that corresponds to the question you
are answering on this page.

Begin your response to each question at the top of a new page. Do not skip lines.

Page 8

Use a pen with black or dark blue ink only. Do NOT write your name. Do NOT write outside the box.

AP Comparative Government & Politics Practice Exam 39


Important: Completely fll in the circle Question 1 Question 2 Question 3 Question 4
that corresponds to the question you
are answering on this page.

Begin your response to each question at the top of a new page. Do not skip lines.

Page 9

Use a pen with black or dark blue ink only. Do NOT write your name. Do NOT write outside the box.

40 AP Comparative Government & Politics Practice Exam


Important: Completely fll in the circle Question 1 Question 2 Question 3 Question 4
that corresponds to the question you
are answering on this page.

Begin your response to each question at the top of a new page. Do not skip lines.

Page 10

Use a pen with black or dark blue ink only. Do NOT write your name. Do NOT write outside the box.

AP Comparative Government & Politics Practice Exam 41


Important: Completely fll in the circle Question 1 Question 2 Question 3 Question 4
that corresponds to the question you
are answering on this page.

Begin your response to each question at the top of a new page. Do not skip lines.

Page 11

Use a pen with black or dark blue ink only. Do NOT write your name. Do NOT write outside the box.

42 AP Comparative Government & Politics Practice Exam


Important: Completely fll in the circle Question 1 Question 2 Question 3 Question 4
that corresponds to the question you
are answering on this page.

Begin your response to each question at the top of a new page. Do not skip lines.

Page 12

Use a pen with black or dark blue ink only. Do NOT write your name. Do NOT write outside the box.

AP Comparative Government & Politics Practice Exam 43


Notes on the AP Comparative
Government and Politics Practice Exam
Multiple-Choice Section
Course Framework Alignment and Rationales

Question 1
Skill Learning Objective Topic
1.B: Explain political systems, PAU-2.A: a. Describe Federal and
principles, institutions, processes, federal and unitary systems Unitary
policies, and behaviors. among course countries. Systems
b. Explain the purposes
of adopting a federal or
unitary system.
(A) Incorrect. Power can be diffused among different levels of government
in a unitary system, but this does not necessarily lead to slow or
contradictory responses to the needs of citizens. A unitary state is one
in which power is concentrated in the national government with some
distribution to subnational levels. This can lead to more uniform policies
and more efficient policy making because most decisions are coming
from one source.
(B) Incorrect. Power is not shared equally between a national government
and subnational governments in a unitary system. A unitary state
is one in which power is concentrated in the national government
with some distribution to subnational levels. Rather than static or
unchanging policies, policies in a unitary state can be more uniform
and more efficient making because most decisions are coming from
one source.
(C) Correct. A unitary state is one in which power is concentrated in the
national government with some distribution to subnational levels. This
can lead to more uniform policies and more efficient policy making
because most decisions are coming from one source.
(D) Incorrect. While the national government is more powerful than
subnational governments in a unitary state, there are still other levels
of government with some powers and responsibilities. A unitary state
is one in which power is concentrated in the national government with
some distribution to subnational levels. This can lead to more uniform
policies and more efficient policy making because most decisions are
coming from one source.

44 AP Comparative Government & Politics Practice Exam


Question 2
Skill Learning Objective Topic
1.A: Describe political systems, MPA-1.A: Explain The Practice
principles, institutions, processes, how political scientists of Political
policies, and behaviors. construct knowledge and Scientists
communicate inferences and
explanations about political
systems, institutional
interactions, and behavior.
(A) Correct. The Gini index measures economic inequality.
(B) Incorrect. HDI measures the standard of living via life expectancy,
literacy, and per capita GDP. It does not assess economic inequality. The
Gini index measures economic inequality.
(C) Incorrect. Per capita GDP measures the average level of production in
a society, not how it is distributed. The Gini index measures economic
inequality.
(D) Incorrect. Freedom House’s civil liberties score measures civil liberties
such as freedom of speech and religion, not economic inequality. The
Gini index measures economic inequality.

Question 3
Skill Learning Objective Topic
1.C: Compare political systems, IEF-2.B: Describe pluralist Pluralist and
principles, institutions, processes, and corporatist interest Corporatist
policies, and behaviors. group systems. Interests
(A) Incorrect. Pluralist systems promote competition and not economic
policies that are developed and run by the state. Pluralism is a system
of interest group representation that promotes competition among
autonomous groups not linked to the state.
(B) Incorrect. Corporatist systems, not pluralist systems, allow the state to
maintain control over citizen input and participation. Pluralist systems,
on the other hand, promote competition among autonomous groups not
linked to the state.
(C) Correct. Pluralism is a system of interest group representation that
promotes competition among autonomous groups not linked to the
state.
(D) Incorrect. Pluralist systems do not use single peak associations to
represent the business sector. Pluralism is a system of interest group
representation that promotes competition among autonomous groups
not linked to the state.

AP Comparative Government & Politics Practice Exam 45


Question 4
Skill Learning Objective Topic
1.D: Describe political systems, DEM-2.B: Explain how Objectives of
principles, institutions, processes,election rules serve different Election Rules
policies, and behaviors of a regime objectives regarding
course country. ballot access, election
wins, and constituency
accountability.
(A) Incorrect. Lords do not earn seats through party list appointments. The
most common way for a Lord to earn a seat is by being nominated by
the prime minister and approved by the monarch.
(B) Incorrect. Members of the House of Commons do not elect Lords. The
most common way for a Lord to earn a seat is by being nominated by
the prime minister and approved by the monarch.
(C) Incorrect. Lords do not win seats through a national election. The most
common way for a Lord to earn a seat is by being nominated by the
prime minister and approved by the monarch.
(D) Correct. The vast majority of Lords are nominated by the prime minister,
working with a special commission, and approved by the monarch.

Question 5
Skill Learning Objective Topic
1.B: Explain political systems, LEG-5.A: Explain how Impact of
principles, institutions, processes, natural resources affect Natural
policies, and behaviors. political and economic Resources
development.
(A) Incorrect. Oil-rich countries typically have overvalued, not undervalued,
currencies. Lack of economic diversification makes them vulnerable to
debt when oil prices fall.
(B) Incorrect. Inequality that results from concentration of wealth in a
narrow part of the economy may make such states prone to social
conflict, but not necessarily economic crises. Lack of economic
diversification makes them vulnerable to debt when oil prices fall.
(C) Correct. Lack of economic diversification makes states prone to
economic crises. If oil prices fall globally, states can be left with too
much debt and not enough income.
(D) Incorrect. Oil-rich countries typically have low taxes, not high taxes,
since governments secure substantial revenue from oil. Lack of
economic diversification makes them vulnerable to debt when oil prices
fall.

46 AP Comparative Government & Politics Practice Exam


Question 6
Skill Learning Objective Topic
2.A: Compare two or more DEM-2.B: Explain Objectives of
course countries based on their how election rules Election Rules
political systems, principles, serve different regime
institutions, processes, policies, objectives regarding
and behaviors. ballot access, election
wins, and constituency
accountability.
(A) Correct. Iran and Russia both require that the chief executive receive an
absolute majority of the public’s votes to be elected, with a second-round
runoff if necessary.
(B) Incorrect. Iran requires an absolute majority for a candidate to become
president, but Mexico only requires that the candidate receive a plurality
of the national popular vote. Iran and Russia both require that the chief
executive receive an absolute majority of the public’s votes to be elected.
(C) Incorrect. Nigeria does require distribution of votes across the states of
the country for a candidate to be elected, but not an absolute majority
overall. The prime minister of the United Kingdom is not directly
elected. Iran and Russia both require that the chief executive receive an
absolute majority of the public’s votes to be elected.
(D) Incorrect. Mexico only requires that the candidate receive a plurality of
the vote to become president. Nigeria does require distribution of votes
across the states of the country for a candidate to be elected, but not an
absolute majority overall. Iran and Russia both require that the chief
executive receive an absolute majority of the public’s votes to be elected.

AP Comparative Government & Politics Practice Exam 47


Question 7
Skill Learning Objective Topic
2.B: Explain the implications of DEM-2.A: Describe Electoral
the similarities and differences electoral systems and Systems and
between countries with similar election rules among course Rules
political systems, principles, countries.
institutions, processes, policies,
and behaviors.
(A) Incorrect. Both countries’ politics and political parties gain support
beyond simple class divisions. Other cleavages are also important, but
the parties aim to appeal to broad constituencies. Both countries use
single-member district electoral systems with first-past-the-post rules
which tend to favor catch-all parties.
(B) Correct. Both countries use single-member district electoral systems
with first-past-the-post rules which tend to favor catch-all parties.
(C) Incorrect. Nigeria has three large ethnic groups which should produce at
least three major parties representing those ethnic cleavages. The United
Kingdom has important regional cleavages, but, like Nigeria, this should
tend to create more than two major parties for the regions; instead, both
parties are competitive across regions and regional parties earn fewer
seats than votes. Both countries use single-member district electoral
systems with first-past-the-post rules which tend to favor catch-all
parties.
(D) Incorrect. Nigeria’s Independent National Electoral Commission is
regarded as making elections fair without suppressing smaller parties.
The United Kingdom does not use proportional representation. If it
did, the likely effect would be to increase the number of parties. Both
countries use single-member district electoral systems with first-past-
the-post rules which tend to favor catch-all parties.

48 AP Comparative Government & Politics Practice Exam


Question 8
Skill Learning Objective Topic
1.E: Explain how political LEG-4.A: Explain political Causes and
systems, principles, institutions, causes and consequences of Effects of
processes, policies, and behaviors demographic changes. Demographic
apply in a course country. Change
(A) Correct. The North American Free Trade Agreement (NAFTA) and
other economic liberalization policies were the stimulus that generated
significant job growth in northern Mexico, attracting migrants from the
south to the north.
(B) Incorrect. Economic liberalization policies did not fail in northern
Mexico. The North American Free Trade Agreement (NAFTA) and
other economic liberalization policies were the stimulus that generated
significant job growth in northern Mexico, attracting migrants from the
south to the north.
(C) Incorrect. Economic liberalization policies were not implemented in
rural areas of Mexico. The North American Free Trade Agreement
(NAFTA) and other economic liberalization policies were the stimulus
that generated significant job growth in northern Mexico, attracting
migrants from the south to the north.
(D) Incorrect. Economic liberalization policies were not implemented
in southern Mexico. The North American Free Trade Agreement
(NAFTA) and other economic liberalization policies were the stimulus
that generated significant job growth in northern Mexico, attracting
migrants from the south to the north.

Question 9
Skill Learning Objective Topic
1.E: Explain how political DEM-1.C: Explain the Civil Rights and
systems, principles, institutions, extent to which civil rights Civil Liberties
processes, policies, and and civil liberties are
behaviors apply in a course protected or restricted in
country. different regimes.
(A) Incorrect. The Great Firewall refers to internet censorship, not
infrastructure. The Great Firewall refers to the policies and technology
used by the Chinese government to severely censor the internet and so
limit criticism on social media.
(B) Incorrect. The Great Firewall refers to policies about internet use,
not migration patterns. The Great Firewall refers to the policies and
technology used by the Chinese government to severely censor the
internet and so limit criticism on social media.
(C) Incorrect. The Great Firewall did not increase global influence through
cross-border road projects. The Great Firewall refers to the policies
and technology used by the Chinese government to severely censor the
internet and so limit criticism on social media.
(D) Correct. The Great Firewall refers to the policies and technology used
by the Chinese government to severely censor the internet and to limit
criticism on social media.

AP Comparative Government & Politics Practice Exam 49


Question 10
Skill Learning Objective Topic
2.A: Compare two or more IEF-3.B: Compare political Political
course countries based on their responses to global market Responses to
political systems, principles, forces. Global Market
institutions, processes, policies, Forces
and behaviors.
(A) Incorrect. While China has privatized most of its economy to promote
competition, Mexico remains a member of the North American Free
Trade Agreement (NAFTA). In an effort to grow their economies and
respond to global market forces, China created special economic zones
(SEZs) and Mexico worked to increase competition in its oil industry.
(B) Incorrect. The United Kingdom has not nationalized the banking
industry. Although the Revolutionary Guard influences the Iranian
economy, it does not directly supervise the economy. In an effort to
grow their economies and respond to global market forces, China
created special economic zones (SEZs) and Mexico worked to increase
competition in its oil industry.
(C) Correct. In an effort to grow their economies and respond to global
market forces, China created special economic zones (SEZs) and Mexico
worked to increase competition in its oil industry.
(D) Incorrect. The United Kingdom has not nationalized natural resources,
but Russia has nationalized its oil and natural gas resources. In an effort
to grow their economies and respond to global market forces, China
created special economic zones (SEZs) and Mexico worked to increase
competition in its oil industry.

Question 11
Skill Learning Objective Topic
1.D: Describe political systems, PAU-3.G: Describe the Judicial Systems
principles, institutions, structure and functions of
processes, policies, and judiciaries.
behaviors of a course country.
(A) Incorrect. Mexico has a functioning Supreme Court. Supreme Court
justices are appointed by the president and confirmed by the Senate.
They serve a term of 15 years.
(B) Incorrect. The Mexican judiciary has the power of judicial review.
Supreme Court justices in Mexico are appointed by the president and
confirmed by the Senate. They serve a term of 15 years.
(C) Correct. Supreme Court justices in Mexico are appointed by the
president and confirmed by the Senate. They serve a term of 15 years.
(D) Incorrect. In Mexico Supreme Court justices are appointed to a term
of 15 years, not a life term. Supreme Court justices in Mexico are
appointed by the president and confirmed by the Senate.

50 AP Comparative Government & Politics Practice Exam


Question 12
Skill Learning Objective Topic
1.A: Describe political systems, IEF-2.B: Describe pluralist Pluralist and
principles, institutions, and corporatist interest Corporatist
processes, policies, and group systems. Interests
behaviors.
(A) Incorrect. Socialism is the belief in the reduction of income disparities
and the nationalization of private industries. “Socialist” does not
describe an interest group system that controls access to policy making
by relying on state-sanctioned groups to represent labor, business,
and agriculture. A corporatist interest group system is one in which
the government controls access to policy making by relying on state-
sanctioned groups or single peak associations to represent labor,
business, and agricultural sectors.
(B) Correct. A corporatist interest group system is one in which the
government controls access to policy making by relying on state-
sanctioned groups or single peak associations to represent labor,
business, and agricultural sectors.
(C) Incorrect. Populism is a political philosophy that supports the interest
and rights of the common people over the elites. A corporatist interest
group system is one in which the government controls access to
policy making by relying on state-sanctioned groups or single peak
associations to represent labor, business, and agricultural sectors.
(D) Incorrect. A pluralist interest group system promotes competition
among autonomous groups not linked to the state. A corporatist
interest group system is one in which the government controls access
to policy making by relying on state-sanctioned groups or single peak
associations to represent labor, business, and agricultural sectors.

AP Comparative Government & Politics Practice Exam 51


Question 13
Skill Learning Objective Topic
3.B: Describe patterns and trends MPA-1.A: Explain The Practice of
in data. how political scientists Political Scien-
construct knowledge and tists
communicate inferences
and explanations
about political systems,
institutional interactions,
and behavior.
(A) Incorrect. The chart shows that life expectancy mostly fell from 1985 to
1991. In 2000, when Putin became president, life expectancy was on a
decline that started in 1998.
(B) Incorrect. The chart does not start until 1985, so it does not show what
happened from 1922 to 1985. In 2000, when Putin became president of
Russia, life expectancy was on a decline that started in 1998.
(C) Incorrect. When Putin first became president of Russia in 2000, life
expectancy fell and then rose. In 2000, when Putin became president,
life expectancy was on a decline that started in 1998.
(D) Correct. In 2000, when Putin became president of Russia, life
expectancy was on a decline that started in 1998.

Question 14
Skill Learning Objective Topic
3.D: Explain what the data LEG-1.B: Explain how Sustaining
implies or illustrates about governments maintain Legitimacy
political systems, principles, legitimacy.
institutions, processes, policies,
and behaviors.
(A) Incorrect. From 2005 to 2015, life expectancy increased. It is unlikely
international aid groups would respond to an increase in life
expectancy with more aid. The most likely political result of increased
life expectancy, especially if it is believed to be a result of government
policies, is that the government’s legitimacy will increase.
(B) Correct. The most likely political result of increased life expectancy,
especially if it is believed to be a result of government policies, is that the
government’s legitimacy will increase.
(C) Incorrect. If the average age of retirement increases, the government is
more likely to receive increased revenues, as there are more workers to
pay taxes. The most likely political result of increased life expectancy,
especially if it is believed to be a result of government policies, is that the
government’s legitimacy will increase.
(D) Incorrect. Elderly people are more likely to vote than young people. The
most likely political result of increased life expectancy, especially if it is
believed to be a result of government policies, is that the government’s
legitimacy will increase.

52 AP Comparative Government & Politics Practice Exam


Question 15
Skill Learning Objective Topic
3.E: Explain possible limitations MPA-1.A: Explain The Practice
of the data provided. how political scientists of Political
construct knowledge and Scientists
communicate inferences
and explanations
about political systems,
institutional interactions,
and behavior.
(A) Incorrect. Because the data are verifiable, they are empirical and not
normative. A significant limitation of the data in the chart is that there
is no distinction between male and female life expectancy, and the
differences are very significant in Russia because women tend to live
much longer than men.
(B) Incorrect. Although the Soviet Union collapsed in 1991, the data for the
Soviet Union and Russia are useful in analyses. One can use the data
to analyze Russia post-1991, the Soviet Union from 1985-1991, or to
make a general comparison between the Soviet Union and Russia over
time. A significant limitation of the data in the chart is that there is no
distinction between male and female life expectancy, and the differences
are very significant in Russia because women tend to live much longer
than men.
(C) Correct. A significant limitation of the data in the chart is that there
is no distinction between male and female life expectancy, and the
differences are very significant in Russia because women tend to live
much longer than men.
(D) Incorrect. There are enough data points to show a clear increase in life
expectancy at birth in Russia between 2005 and 2017. A significant
limitation of the data in the chart is that there is no distinction between
male and female life expectancy, and the differences are very significant
in Russia because women tend to live much longer than men.

AP Comparative Government & Politics Practice Exam 53


Question 16
Skill Learning Objective Topic
2.A: Compare two or more PAU-3.F: Explain how Independent
course countries based on their legislative powers are Legislatures
political systems, principles, constrained by other
institutions, processes, policies, institutions and/or
and behaviors. processes, which can affect
legislative independence.
(A) Incorrect. Both the INE and INEC ensure electoral fairness, and do not
participate in the legislative process. China’s Standing Committee of the
Politburo and Iran’s Guardian Council both have substantial influence
on the legislative process.
(B) Correct. Both the Standing Committee and the Guardian Council
have the ability to influence the legislative agenda and to steer or alter
decisions of the legislature.
(C) Incorrect. The United Kingdom’s House of Lords reviews and amends
bills, but does not interfere with the House of Commons’ legislative
independence, and the Nigerian legislature has constitutional
protections to maintain its independence from the president. China’s
Standing Committee of the Politburo and Iran’s Guardian Council both
have substantial influence on the legislative process.
(D) Incorrect. Iran’s Supreme Leader has substantial influence on the
legislative process, but the British monarch has almost none. China’s
Standing Committee of the Politburo and Iran’s Guardian Council both
have substantial influence on the legislative process.

54 AP Comparative Government & Politics Practice Exam


Question 17
Skill Learning Objective Topic
1.C: Compare political systems, PAU-3.A: Describe Parliamentary,
principles, institutions, processes, parliamentary, Presidential,
policies, and behaviors. presidential, and semi- and Semi-
presidential systems. Presidential
Systems
(A) Incorrect. While a head of government often serves as the leader of
the legislature with sole responsibility for creating policy and passing
laws, a head of state does not normally serve as the chief executive
responsible for enforcing laws and treaties. A head of state generally
serves as the symbolic political representative of the nation while a head
of government is the official responsible for running the government.
(B) Incorrect. A head of government is often in charge of foreign policy,
but usually not the sole leader in charge of it. A head of state does not
normally conduct domestic policy. A head of state generally serves
as the symbolic political representative of the nation while a head of
government is the official responsible for running the government.
(C) Incorrect. A head of state, not head of government, generally serves
as the symbolic political representative of the nation. The head of
government, not head of state, is the official responsible for running the
government.
(D) Correct. A head of state generally serves as the symbolic political
representative of the nation while a head of government is the official
responsible for running the government.

Question 18
Skill Learning Objective Topic
1.B: Explain political systems, PAU-3.C: Explain the Executive Term
principles, institutions, processes, structure, function, and Limits
policies, and behaviors. change of executive
leadership in course
countries.
(A) Incorrect. Limiting time in office does not allow legislators to develop
coherent policies over time. Limiting the time in office inhibits the
ability of executives to consolidate power.
(B) Incorrect. Limiting the time in office does not allow executive greater
control over the legislative process. Limiting the time in office inhibits
the ability of executives to consolidate power.
(C) Correct. By limiting the time in office, executive term limits can prevent
executives from consolidating power.
(D) Incorrect. Limiting the time in office is not likely to weaken
accountability and is not an advantage of executive term limits. Limiting
the time in office can inhibit the ability of executives to consolidate
power.

AP Comparative Government & Politics Practice Exam 55


Question 19
Skill Learning Objective Topic
2.C: Explain the implications of LEG-2.B: Explain how Political and
the similarities and differences political and social Social Cleavages
between countries with different cleavages in course
political systems, principles, countries affect citizen
institutions, processes, policies, relationships and political
and behaviors. stability.
(A) Incorrect. Regional cleavages have not led to groups successfully gaining
independence from Iran. In the United Kingdom groups in Scotland
have demanded greater autonomy and even held a referendum on
independence. In Mexico, Zapatistas in Chiapas have only demanded
greater autonomy.
(B) Incorrect. In Nigeria, groups such as Boko Haram have formed
separatist movements because of religious cleavages, resulting in a
response from the military. However, in Mexico, religious cleavages
are not significant and have not led to a separatist movement. In the
United Kingdom regional cleavages have resulted in demands for
greater autonomy and independence, whereas in Mexico they have only
resulted in demands for autonomy.
(C) Correct. In the United Kingdom groups in Scotland have demanded
greater autonomy and even held a referendum on independence. In
Mexico, Zapatistas in Chiapas have demanded only greater autonomy.
(D) Incorrect. Although there has been ethnic conflict in China and
Russia, neither state has given fair representation to these groups. In
the United Kingdom regional cleavages have resulted in demands for
greater autonomy and independence, whereas in Mexico they have only
resulted in demands for autonomy.

56 AP Comparative Government & Politics Practice Exam


Question 20
Skill Learning Objective Topic
2.A: Compare two or more LEG-1.A: Describe Political
course countries based on their the sources of political Legitimacy
political systems, principles, legitimacy for different
institutions, processes, policies, types of regimes among
and behaviors. course countries.
(A) Incorrect. While in Mexico rule of law is one source of legitimacy,
in Iran, the regime does not derive its legitimacy from the Assembly
of Experts. In Iran, the regime primarily derives its legitimacy from
religion, while in Mexico the primary source of legitimacy is the
constitution.
(B) Incorrect. In Iran, the regime does not primarily derive its legitimacy
from the rule of law, while in Mexico, the primary source of legitimacy
is not the Supreme Court. In Iran, the regime derives its legitimacy
from religion, while in Mexico the primary source of legitimacy is the
constitution.
(C) Incorrect. In Iran, the regime does not derive its legitimacy from the
supreme leader, while in Mexico the primacy source of legitimacy is not
the president. In Iran, the regime derives its legitimacy from religion,
while in Mexico the primary source of legitimacy is the constitution.
(D) Correct. In Iran, the regime primarily derives its legitimacy from
religion, while in Mexico the primary source of legitimacy is the
constitution.

Question 21
Skill Learning Objective Topic
1.E: Explain how political IEF-3.E: Explain Policies and
systems, principles, institutions, the adoption of and Economic
processes, policies, and behaviors consequences associated Liberalization
apply in a course country. with economic
liberalization policies.
(A) Correct. Economic liberalization in China created job growth on the
east coast, causing migration from the west to the east.
(B) Incorrect. Economic liberalization in China created job growth in urban
areas, primarily on the east coast. This drove migration from the west to
the east, not east to the west.
(C) Incorrect. Economic liberalization in China created job growth in urban
areas, driving migration from the countryside to the cities. Economic
liberalization in China created job growth on the east coast, causing
migration from the west to the east.
(D) Incorrect. Economic liberalization in China created job growth in the
east, not north. Economic liberalization in China created job growth on
the east coast, causing migration from the west to the east.

AP Comparative Government & Politics Practice Exam 57


Question 22
Skill Learning Objective Topic
2.A: Compare two or more PAU-4.B: Explain how Role of Political
course countries based on their political party systems and Party Systems
political systems, principles, memberships link citizen
institutions, processes, policies, participation to policy-
and behaviors. making.
(A) Incorrect. While Russia has a dominant-party system, China has a one-
party system, not a two-party system.
(B) Correct. China has a one-party system while Russia has a dominant-
party system.
(C) Incorrect. China has a one-party system, not a multi-party system.
Russia has a dominant-party system, not a one-party system.
(D) Incorrect. China has a one-party system, not a dominant-party system.
Russia has a dominant-party system, not a one-party system.

Question 23
Skill Learning Objective Topic
4.A: Describe the author’s IEF-2.A: Explain how Executive
claim(s), perspective, evidence, social movements and Systems
and reasoning. interest groups affect social
and political change.
(A) Incorrect. The authors do say that Khamenei changed the role of the
president, but this is not used as evidence that Khamenei sided with
Ahmadinejad. Khamenei did use the Revolutionary Guard to preserve
the status quo by suppressing protests.
(B) Correct. Khamenei did use the Revolutionary Guard to preserve the
status quo by suppressing protests.
(C) Incorrect. The authors do not assert that Khamenei personally altered
the results of the election. Khamenei did use the Revolutionary Guard to
preserve the status quo by suppressing protests.
(D) Incorrect. The authors do not mention the Green Movement by name.
The Green Movement was an opposition movement in response
to the elections and not part of the regime. Khamenei did use the
Revolutionary Guard to preserve the status quo by suppressing protests.

58 AP Comparative Government & Politics Practice Exam


Question 24
Skill Learning Objective Topic
4.B: Explain how the author’s PAU-3.C: Explain the Executive
argument or perspective relates structure, function, and Systems
to political systems, principles, change of executive
institutions, processes, policies, leadership in course
and behaviors. countries.
(A) Incorrect. The supreme leader may set the political agenda in his role
as chief legislator, but this does not explain why Khamenei has the
authority to take action as described by the authors. The supreme
leader controls the Revolutionary Guards as commander in chief, and
Khamenei ordered the Revolutionary Guards to preserve the status quo.
(B) Incorrect. The supreme leader does not serve as the head of government;
that responsibility belongs to the president. The supreme leader controls
the Revolutionary Guards as commander in chief, and Khamenei
ordered the Revolutionary Guards to preserve the status quo.
(C) Correct. The supreme leader controls the Revolutionary Guards as
commander in chief, and Khamenei ordered the Revolutionary Guards
to preserve the status quo.
(D) Incorrect. Khamenei is not the chief justice of Iran, which is a title
given to the head of the judiciary. The judiciary did not play a role
in suppressing the election protests. The supreme leader controls the
Revolutionary Guards as commander in chief, and Khamenei ordered
the Revolutionary Guards to preserve the status quo.

AP Comparative Government & Politics Practice Exam 59


Question 25
Skill Learning Objective Topic
4.C: Explain how the implications LEG-1.B: Explain how Sustaining
of the author’s argument or governments maintain Legitimacy
perspective may affect political legitimacy.
systems, principles, institutions,
processes, policies, and behaviors.
(A) Correct. Legitimacy is the people’s belief that the regime has the right to
rule. The authors imply that Khamenei’s use of force to counter protest
will undermine that belief.
(B) Incorrect. The authors do not mention outside powers’ disapproval
of the regime’s electoral practices. The authors imply that the regime’s
legitimacy will weaken because of the supreme leader’s change from his
traditionally neutral role in politics, which will undermine public trust
in the leadership’s right to rule.
(C) Incorrect. The authors claim that the public favors the protestors against
fraud in the election. The popularity of the Green Movement is also
evidence of this. The authors imply that the regime’s legitimacy will
weaken because of the supreme leader’s change from his traditionally
neutral role in politics, which will undermine public trust in the
leadership’s right to rule.
(D) Incorrect. Coerced obedience does not lead to stronger legitimacy of
a regime. The authors imply that the regime’s legitimacy will weaken
because of the supreme leader’s change from his traditionally neutral
role in politics, which will undermine public trust in the leadership’s
right to rule.

60 AP Comparative Government & Politics Practice Exam


Question 26
Skill Learning Objective Topic
1.B: Explain political systems, PAU-1.C: Explain the Democratization
principles, institutions, processes, process and goals of
policies, and behaviors. democratization.
(A) Incorrect. In some circumstances, such as when harm is inflicted upon
others, groups and individuals in a democracy will have their civil
liberties restricted. However, successful consolidating democracies
focus on protecting the civil rights and liberties of groups or individuals,
not restricting them. Competitive and fair elections are important to
the establishment and the consolidation of a democracy because they
give citizens the right to express their political opinions and to remove
officials from political office.
(B) Incorrect. Establishing rule of law, not rule by law, means government
officials are treated like everyone else. Treating all equally with no one
above the law is important to democratic consolidation. Competitive
and fair elections are important to the establishment and the
consolidation of a democracy because they give citizens the right to
express their political opinions and to remove officials from political
office.
(C) Incorrect. Increasing, not reducing, public access to government
documents allows for greater efficiency and transparency in decision
making processes; greater transparency is important to democratic
consolidation. Competitive and fair elections are important to the
establishment and the consolidation of a democracy because they give
citizens the right to express their political opinions and to remove
officials from political office.
(D) Correct. Competitive and fair elections are important to the
establishment and the consolidation of a democracy because they give
citizens the right to express their political opinions and to remove
officials from political office.

AP Comparative Government & Politics Practice Exam 61


Question 27
Skill Learning Objective Topic
1.C: Compare political systems, LEG-3.A: Explain International
principles, institutions, processes, how international and and
policies, and behaviors. supranational organizations Supranational
influence domestic policy- Organizations
makers and national
sovereignty.
(A) Incorrect. It is true that import substitution policies protect domestic
industries, but neoliberal policies do not seek to decrease FDI. Import
substitution policies support increasing tariffs, and neoliberal policies
support lowering tariffs.
(B) Incorrect. Import substitution policies are generally adopted by
developing countries rather than well-developed countries. Neoliberal
policies are adopted by countries seeking growth, which may or may
not have high levels of inequality. Import substitution policies support
increasing tariffs, and neoliberal policies support lowering tariffs.
(C) Incorrect. Import substitution policies do not necessarily address
government deficits, and neoliberal policies support lowering taxes
rather than raising taxes. Import substitution policies support increasing
tariffs, and neoliberal policies support lowering tariffs.
(D) Correct. Import substitution policies support increasing tariffs, and
neoliberal policies support lowering tariffs.

62 AP Comparative Government & Politics Practice Exam


Question 28
Skill Learning Objective Topic
1.C: Compare political systems, IEF-1.C: Explain how Political
principles, institutions, processes, political culture relates to Ideologies
policies, and behaviors. citizen behavior and the
role of the state.
(A) Incorrect. Populism, not fascism, is the belief in the interests and
rights of the common people over that of the elites. Socialism, not
communism, is the belief in the nationalization of major private
industries. Fascism is the belief in a nationalist ideology favoring
authoritarian rule, and communism is the belief in the removal of
private property and near total government control over the economy.
(B) Correct. Fascism is the belief in a nationalist ideology favoring
authoritarian rule, and communism is the belief in the removal of
private property and near total government control over the economy.
(C) Incorrect. Socialism, not fascism, is the belief in the reduction of
income disparities among all people. Individualism, not communism,
is the belief in individual civil liberties and freedoms over government
intrusion or restriction. Fascism is the belief in a nationalist ideology
favoring authoritarian rule, and communism is the belief in the removal
of private property and near total government control over the economy.
(D) Incorrect. Neoliberalism, not fascism, is the belief in limited
government intervention in the economy and society. Neoliberalism,
not communism, is also the belief in the deregulation of trade and the
elimination of subsidies. Fascism is the belief in a nationalist ideology
favoring authoritarian rule, and communism is the belief in the removal
of private property and near total government control over the economy.

AP Comparative Government & Politics Practice Exam 63


Question 29
Skill Learning Objective Topic
1.E: Explain how political DEM-2.B: Explain how Objectives of
systems, principles, institutions, election rules serve Election Rules
processes, policies, and different regime objectives
behaviors apply in a course regarding ballot access,
country. election wins, and
constituency accountability.
(A) Incorrect. Iran does not have major political parties but rather,
presidential candidates who attempt to create strong factions. The
Guardian Council has the power to vet candidates, and most often
excludes reform-minded candidates or those who do not support
Islamic values from the ballot. This limits the number of candidates and
reduces electoral competition.
(B) Incorrect. The Guardian Council allows nonclerics to run for president.
The Guardian Council has the power to vet candidates and most often
excludes reform-minded candidates or those who do not support
Islamic values from the ballot. This limits the number of candidates and
reduces electoral competition.
(C) Incorrect. The president in Iran is directly elected in a national popular
election. The Guardian Council has the power to vet candidates and
most often excludes reform-minded candidates or those who do not
support Islamic values from the ballot. This limits the number of
candidates and reduces electoral competition.
(D) Correct. The Guardian Council has the power to vet candidates and
most often excludes reform-minded candidates or those who do not
support Islamic values from the ballot. This limits the number of
candidates and reduces electoral competition.

64 AP Comparative Government & Politics Practice Exam


Question 30
Skill Learning Objective Topic
1.B: Explain political systems, LEG-3.A: Explain International
principles, institutions, how international and and
processes, policies, and supranational organizations Supranational
behaviors. influence domestic policy- Organizations
makers and national
sovereignty.
(A) Incorrect. A country would implement an import substitution policy to
protect its own industries rather than having increased influence from
foreign companies. To bolster their own developing industries, countries
implement import substitution industrialization (ISI) policies aimed at
reducing foreign dependency by raising tariffs and encouraging local
production of industrialized products.
(B) Incorrect. Import substitution industrialization policies often increase
tariffs rather than decrease them to protect domestic industries. To
bolster developing industries, countries implement import substitution
industrialization (ISI) policies aimed at reducing foreign dependency
by raising tariffs and encouraging local production of industrialized
products.
(C) Correct. To bolster their own developing industries, countries
implement import substitution industrialization (ISI) policies aimed at
reducing foreign dependency by raising tariffs and encouraging local
production of industrialized products.
(D) Incorrect. The goal of implementing import substitution
industrialization policies is to not to create more state-owned
companies. To bolster their own developing industries, countries
implement import substitution industrialization (ISI) policies aimed at
reducing foreign dependency by raising tariffs and encouraging local
production of industrialized products.

AP Comparative Government & Politics Practice Exam 65


Question 31
Skill Learning Objective Topic
2.C: Explain the implications of PAU-3.G: Describe the Judicial Systems
the similarities and differences structure and functions of
between countries with different judiciaries.
political systems, principles,
institutions, processes, policies,
and behaviors.
(A) Incorrect. Judicial review can only be exercised by independent
judiciaries. The Russian constitution does establish judicial review,
but courts are not independent. Independent courts are critical to
maintaining rule of law.
(B) Incorrect. Rule of law may be maintained in either parliamentary
or semi-presidential systems. Independent courts are critical to
maintaining rule of law. Russia does not have independent courts,
whereas the United Kingdom does.
(C) Incorrect. Both countries have supreme courts. Supreme courts do not
necessarily indicate the presence of rule of law. Russia does not have
independent courts, whereas the United Kingdom does.
(D) Correct. Independent courts are critical to maintaining rule of law.
Russia does not have independent courts, whereas the United Kingdom
does.

66 AP Comparative Government & Politics Practice Exam


Question 32
Skill Learning Objective Topic
2.C: Explain the implications of PAU-4.B: Explain how Objectives of
the similarities and differences political party systems and Election Rules
between countries with different memberships link citizen
political systems, principles, participation to policy
institutions, processes, policies, making.
and behaviors.
(A) Incorrect. There is no effective political party system in Iran. The
political party system in the United Kingdom is not corporatist. Political
parties in Iran are unable to effectively support candidates for election
and are not well-established; while in the United Kingdom political
parties are free to compete and the election rules have resulted in a two-
party dominant system.
(B) Incorrect. As a result of the candidate eligibility vetting conducted by
the Guardian Council, political parties are largely non-existent; while in
the United Kingdom political parties have clear platforms, membership
rules and compete effectively in elections. Political parties in Iran are
unable to effectively support candidates for election and are not well-
established; while in the United Kingdom political parties are free to
compete and the election rules have resulted in a two-party dominant
system.
(C) Correct. As a result of the candidate eligibility vetting conducted by
the Guardian Council, political parties in Iran are unable to effectively
support candidates for election and are not well-established; while in the
United Kingdom political parties are free to compete and the election
rules have resulted in a two-party dominant system.
(D) Incorrect. Fundraising has not impacted the party systems of either
Iran or the United Kingdom. As a result of the candidate eligibility
vetting conducted by the Guardian Council, political parties in Iran are
unable to effectively support candidates for election and are not well-
established; while in the United Kingdom political parties are free to
compete and the election rules have resulted in a multiparty system with
two major parties.

AP Comparative Government & Politics Practice Exam 67


Question 33
Skill Learning Objective Topic
2.A: Compare two or more DEM-1.C: Explain the Civil Rights and
course countries based on their extent to which civil rights Civil Liberties
political systems, principles, and civil liberties are
institutions, processes, policies, protected or restricted in
and behaviors. different regimes.
(A) Incorrect. Russia exercises significant state control and ownership of the
media, whereas Nigeria has a relatively free press. In both China and
Russia, the government exercises significant control of the media.
(B) Incorrect. Iran exercises significant state control of the media, whereas
Mexico has a relatively free press. In both China and Russia, the
government exercises significant control of the media.
(C) Correct. In both China and Russia, the government exercises significant
control of the media.
(D) Incorrect. In both the United Kingdom and Nigeria, there is a relatively
free press. In both China and Russia, the government exercises
significant control of the media.

Question 34
Skill Learning Objective Topic
1.B: Explain political systems, IEF-3.C: Explain how Challenges
principles, institutions, globalization creates from
processes, policies, and challenges to regime Globalization
behaviors. sovereignty.
(A) Incorrect. Multinational corporations often infringe on the sovereignty
of national governments by not obeying local laws and by challenging
power structures. A consequence of globalization is that it often leads
to increased social tensions, as it tends to challenge traditional power
structures and culture.
(B) Incorrect. Globalization may have an indirect impact on regimes, but
does not usually result in regime instability caused by international
pressures for democratization. A consequence of globalization is
that it often leads to increased social tensions, as it tends to challenge
traditional power structures and culture.
(C) Correct. Globalization includes the spread of ideas, technology,
information, and jobs across international borders. A consequence is
that globalization often leads to increased social tensions, as it tends to
challenge traditional power structures and culture.
(D) Incorrect. Importing cheaper goods from advanced industrial
economies does not always lead to a reduction in economic inequality.
A consequence of globalization is that it often leads to increased social
tensions, as it tends to challenge traditional power structures and
culture.

68 AP Comparative Government & Politics Practice Exam


Question 35
Skill Learning Objective Topic
3.A: Describe the data presented. MPA-1.A: Explain The Practice
how political scientists of Political
construct knowledge and Scientists
communicate inferences
and explanations about
political systems,
institutional interactions,
and behavior.
(A) Incorrect. A total of 38% of respondents believed the statement “Elected
officials care what ordinary people think” described Nigeria well.
In response to the question, “How well does the statement describe
Nigeria,” the statement “Most politicians are corrupt” received the
highest percentage (72% of respondents).
(B) Incorrect. A total of 57% of respondents believed the statement
“No matter who wins an election, things do not change very much”
described Nigeria well. In response to the question, “How well does the
statement describe Nigeria,” the statement “Most politicians are corrupt”
received the highest percentage (72% of respondents).
(C) Incorrect. A total of 43% of respondents believed the statement “The
court system treats everyone fairly” described Nigeria well. In response
to the question, “How well does the statement describe Nigeria,” the
statement “Most politicians are corrupt” received the highest percentage
(72% of respondents).
(D) Correct. In response to the question, “How well does the statement
describe Nigeria,” the statement “Most politicians are corrupt” received
the highest percentage (72% of respondents) compared to the other
statements in the poll.

AP Comparative Government & Politics Practice Exam 69


Question 36
Skill Learning Objective Topic
3.D: Explain what the data PAU-3.G: Describe the Judicial Systems
implies or illustrates about structure and functions of
political systems, principles, judiciaries.
institutions, processes, policies,
and behaviors.
(A) Incorrect. Under the system of federalism, Islamic Shariah courts have
been established in the north. An effort has been made to reestablish
the legitimacy of the Nigerian judicial system by protecting its power of
judicial review and independence by reducing corruption.
(B) Incorrect. The Nigerian government has not arbitrarily arrested or
issued lengthy sentences to political protesters. An effort has been
made to reestablish the legitimacy of the Nigerian judicial system by
protecting its power of judicial review and independence by reducing
corruption.
(C) Incorrect. Although the president has the power to appoint judges, the
president has not been significantly increasing the size of the Nigerian
court system. An effort has been made to reestablish the legitimacy of
the Nigerian judicial system by protecting its power of judicial review
and independence by reducing corruption.
(D) Correct. An effort has been made to reestablish the legitimacy of the
Nigerian judicial system by protecting its power of judicial review and
independence by reducing corruption.

70 AP Comparative Government & Politics Practice Exam


Question 37
Skill Learning Objective Topic
3.D: Explain what the data LEG-4.A: Explain political Causes and
implies or illustrates about causes and consequences of Effects of
political systems, principles, demographic changes. Demographic
institutions, processes, policies, Change
and behaviors.
(A) Incorrect. The Nigerian independent election commission has helped to
reduce voter fraud and has provided transparency to Nigerian elections.
Nigerian citizens are critical of Nigerian politicians and have low levels
of political trust, as demonstrated by the majority of respondents who
say that “no matter who wins an election, things do not change very
much” and “most politicians are corrupt.”
(B) Incorrect. Civil society does exist in Nigeria and has been able to
challenge corrupt politicians. Nigerian citizens are critical of Nigerian
politicians and have low levels of political trust, as demonstrated by the
majority of respondents who say that “no matter who wins an election,
things do not change very much” and “most politicians are corrupt.”
(C) Correct. Nigerian citizens are critical of Nigerian politicians and
have low levels of political trust, as demonstrated by the majority of
respondents who say that “no matter who wins an election, things do
not change very much” and “most politicians are corrupt.”
(D) Incorrect. The federalism system in Nigeria has allowed for the north to
have Shariah Islamic courts that address specific needs of that region.
Nigerian citizens are critical of Nigerian politicians and have low levels
of political trust, as demonstrated by the majority of respondents who
say that “no matter who wins an election, things do not change very
much” and “most politicians are corrupt.”

AP Comparative Government & Politics Practice Exam 71


Question 38
Skill Learning Objective Topic
1.A: Describe political systems, DEM-1.C: Explain the Civil Rights and
principles, institutions, extent to which civil rights Civil Liberties
processes, policies, and and civil liberties are
behaviors. protected or restricted in
different regimes.
(A) Correct. Competitive authoritarian regimes are sometimes referred
to as illiberal democracies since they hold contested elections but with
limited degrees of competitiveness and minimal civil liberty protections.
(B) Incorrect. Substantive democracies are those that hold competitive
elections and have strong civil liberties protections. Competitive
authoritarian regimes are sometimes referred to as illiberal democracies
since they hold contested elections but with limited degrees of
competitiveness and minimal civil liberty protections.
(C) Incorrect. Although illiberal democracies may be considered weak
democracies as elections are not fully free and civil liberties are
restricted, they are not known as corporatist democracies. Competitive
authoritarian regimes are sometimes referred to as illiberal democracies
since they hold contested elections but with limited degrees of
competitiveness and minimal civil liberty protections.
(D) Incorrect. Democratic socialist regimes are not illiberal democracies.
Competitive authoritarian regimes are sometimes referred to as illiberal
democracies since they hold contested elections but with limited degrees
of competitiveness and minimal civil liberty protections.

72 AP Comparative Government & Politics Practice Exam


Question 39
Skill Learning Objective Topic
2.A: Compare two or more PAU-3.D: Describe Removal of
course countries based on their procedures for the removal Executives
political systems, principles, of executive leadership by
institutions, processes, policies, other institutions.
and behaviors.
(A) Incorrect. The House of Commons in the United Kingdom does not
have the power of impeachment; it has a no-confidence procedure
instead. The Chamber of Deputies in Mexico does have the power of
impeachment. The Duma in Russia and the House of Representatives in
Nigeria have the power of impeachment.
(B) Incorrect. The National People’s Congress in China does not have
the power of impeachment. The Duma in Russia and the House of
Representatives in Nigeria have the power of impeachment.
(C) Correct. The constitution in Russia grants the Duma the power
of impeachment. The constitution in Nigeria grants the House of
Representatives the power of impeachment.
(D) Incorrect. The Majles in Iran does have the power of impeachment. The
House of Commons in the United Kingdom does not have the power of
impeachment; it has a no-confidence procedure instead. The Duma in
Russia and the House of Representatives in Nigeria have the power of
impeachment.

AP Comparative Government & Politics Practice Exam 73


Question 40
Skill Learning Objective Topic
2.B: Explain the implications of PAU-3.E: Describe Legislative
the similarities and differences legislative structures Systems
between countries with similar and functions in course
political systems, principles, countries.
institutions, processes, policies,
and behaviors.
(A) Incorrect. The United Kingdom is a unitary state, so subnational
governments are subordinate to the central government. Mexico is
a federal state, and its upper house does have the unique power of
approval regarding federal interventions in state matters. However, in
both countries, both chambers participate in policy making, which
slows the process down.
(B) Incorrect. The House of Lords, which is the upper house in the United
Kingdom, does not have the power to approve troop deployment.
However, in both countries, both chambers participate in policy
making, which slows the process down.
(C) Incorrect. Fusion of power refers to a relation between branches of
government, not chambers of the legislature. However, in both countries,
both chambers participate in policy making, which slows the process
down.
(D) Correct. In the United Kingdom, the House of Lords has the power to
review, debate, and revise legislation. In Mexico both chambers have
legislative powers and so are involved in the policy-making process. By
dividing a legislature into two chambers, policy making will be slowed.

74 AP Comparative Government & Politics Practice Exam


Question 41
Skill Learning Objective Topic
2.A: Compare two or more PAU-3.A: Describe Parliamentary,
course countries based on their parliamentary, presidential, Presidential,
political systems, principles, and semi-presidential and Semi-
institutions, processes, policies, systems. Presidential
and behaviors. Systems
(A) Incorrect. In both Mexico and Nigeria, the courts do not review cabinet
member nominations by the executive. Cabinet members in both
countries are nominated by the executive and must be approved by the
legislature.
(B) Correct. Cabinet members in both countries are nominated by the
executive and must be approved by the legislature.
(C) Incorrect. Cabinet members do not concurrently serve in the legislature.
Cabinet members in both countries are nominated by the executive and
must be approved by the legislature.
(D) Incorrect. Although the cabinet is primarily accountable to the
executive, because members can be impeached, the legislature can
also hold them accountable. Cabinet members in both countries are
nominated by the executive and must be approved by the legislature.

AP Comparative Government & Politics Practice Exam 75


Question 42
Skill Learning Objective Topic
4.A: Describe the author’s MPA-1.A: Explain The Practice
claim(s), perspective, evidence, how political scientists of Political
and reasoning. construct knowledge and Scientists
communicate inferences
and explanations about
political systems,
institutional interactions,
and behavior.
(A) Incorrect. The article explains that the Chinese approach has been
“multi-pronged,” not singular. The article claims that much of the
Chinese financing is through large loans to African governments, which
they will need to pay back in the future, raising the chances of a debt
crisis.
(B) Incorrect. The article refers to Chinese banks, contractors, and the
government giving loans to Africa. The article claims that much of the
Chinese financing is through large loans to African governments, which
they will need to pay back in the future, raising the chances of a debt
crisis.
(C) Incorrect. The author does not compare development loans to other
revenue sources. The article claims that much of the Chinese financing
is through large loans to African governments, which they will need to
pay back in the future, raising the chances of a debt crisis.
(D) Correct. The article claims that much of the Chinese financing is
through large loans to African governments, which they will need to pay
back in the future, raising the chances of a debt crisis.

76 AP Comparative Government & Politics Practice Exam


Question 43
Skill Learning Objective Topic
4.C: Explain how the IEF-3.C: Explain how Challenges
implications of the author’s globalization creates from
argument or perspective challenges to regime Globalization
may affect political systems, sovereignty.
principles, institutions,
processes, policies, and
behaviors.
(A) Incorrect. China itself does not strictly adhere to Communist principles,
so it is unlikely that African nations would attempt to attract aid
with such policies. Indebted African nations may lose sovereignty to
the Chinese government and large banks. Foreign direct investment
development loans and the creation of special economic zones are
aspects of globalization that can challenge regime sovereignty.
(B) Correct. Indebted African nations may lose sovereignty to the Chinese
government and large banks. Foreign direct investment development
loans and the creation of special economic zones are aspects of
globalization that can challenge regime sovereignty.
(C) Incorrect. It is unlikely that regional African economic organizations
will offer membership to China. Indebted African nations may lose
sovereignty to the Chinese government and large banks. Foreign direct
investment development loans and the creation of special economic
zones are aspects of globalization that can challenge regime sovereignty.
(D) Incorrect. China’s economic growth is not going to become overly
dependent on resources from Africa. Indebted African nations may lose
sovereignty to the Chinese government and large banks. Foreign direct
investment development loans and the creation of special economic
zones are aspects of globalization that can challenge regime sovereignty.

AP Comparative Government & Politics Practice Exam 77


Question 44
Skill Learning Objective Topic
1.A: Describe political systems, MPA-1.A: Explain The Practice
principles, institutions, how political scientists of Political
processes, policies, and construct knowledge and Scientists
behaviors. communicate inferences
and explanations about
political systems,
institutional interactions,
and behavior.
(A) Incorrect. The Gini index measures income or wealth distribution. The
Freedom House civil liberties score measures the condition of civil
liberties in each nation.
(B) Incorrect. The HDI ranks nations in terms of life expectancy, access
to education, and GDP per capita. The Freedom House civil liberties
score measures the condition of civil liberties in each nation.
(C) Incorrect. The GDP of a nation is its total productivity and measures
the nation’s overall wealth. The Freedom House civil liberties score
measures the condition of civil liberties in each nation.
(D) Correct. The Freedom House civil liberties score measures the
condition of civil liberties in each nation.

Question 45
Skill Learning Objective Topic
2.A: Compare two or more PAU-2.A: a. Describe Federal and
course countries based on their federal and unitary systems Unitary
political systems, principles, among course countries. Systems
institutions, processes, policies, b. Explain the purposes
and behaviors. of adopting a federal or
unitary system.
(A) Correct. Both China and Iran have unitary systems of government,
which concentrate power at the national level with more uniform
policies and potentially more efficient policy-making.
(B) Incorrect. Iran lacks formal political party structures, though China
is a one-party state. Both China and Iran have unitary systems of
government, which concentrate power at the national level with more
uniform policies and potentially more efficient policy-making.
(C) Incorrect. Both China and Iran have a unicameral, not bicameral,
legislature. Both China and Iran have unitary systems of government,
which concentrate power at the national level with more uniform
policies and potentially more efficient policy-making.
(D) Incorrect. Neither China nor Iran have semi-presidential systems. Both
China and Iran have unitary systems of government, which concentrate
power at the national level with more uniform policies and potentially
more efficient policy-making.

78 AP Comparative Government & Politics Practice Exam


Question 46
Skill Learning Objective Topic
2.A: Compare two or more IEF-1.B: Explain the role of Civil Society
course countries based on their civil society among course
political systems, principles, countries.
institutions, processes, policies,
and behaviors.
(A) Incorrect. Both the United Kingdom and Russia may require a
particular group to register with the government. Both authoritarian
and democratic regimes restrict participation and monitor at least some
civil society groups to a certain extent. Russian government monitors
civil society groups much more closely than does the government of the
United Kingdom.
(B) Incorrect. Both the United Kingdom and Russia provide government
funding for some groups. Both authoritarian and democratic regimes
restrict participation and monitor at least some civil society groups to a
certain extent. Russian government monitors civil society groups much
more closely than does the government of the United Kingdom.
(C) Correct. Both authoritarian and democratic regimes restrict
participation and monitor at least some civil society groups to a certain
extent. Russian government monitors civil society groups much more
closely than does the government of the United Kingdom.
(D) Incorrect. The United Kingdom may jail members of opposition groups,
but it does not do so frequently. Both authoritarian and democratic
regimes restrict participation and monitor at least some civil society
groups to a certain extent. Russian government monitors civil society
groups much more closely than does the government of the United
Kingdom.

AP Comparative Government & Politics Practice Exam 79


Question 47
Skill Learning Objective Topic
1.E: Explain how political DEM-1.B: Explain how Forces that
systems, principles, institutions, political participation Impact Political
processes, policies, and behaviors affects and is affected by Participation
apply in a course country. democratic or authoritarian
regime types.
(A) Incorrect. The Chinese government does not promote democratization
among its citizens. The Chinese government allows political protests
to act as a release valve for the public. If there were no outlets for
discontent, there might be even larger protests that could threaten the
regime.
(B) Incorrect. The Chinese government does not purposefully open political
discourse. The Chinese government allows political protests to act as a
release valve for the public. If there were no outlets for discontent, there
might be even larger protests that could threaten the regime.
(C) Incorrect. The Chinese government spreads political information,
but this is not why it allows protests. The Chinese government allows
political protests to act as a release valve for the public. If there were no
outlets for discontent, there might be even larger protests that could
threaten the regime.
(D) Correct. The Chinese government allows political protests to act as a
release valve for the public. If there were no outlets for discontent, there
might be even larger protests that could threaten the regime.

Question 48
Skill Learning Objective Topic
3.B: Describe patterns and MPA-1.A: Explain The Practice
trends in data. how political scientists of Political
construct knowledge and Scientists
communicate inferences
and explanations about
political systems,
institutional interactions,
and behavior.
(A) Incorrect. The percentage of females with primary school education in
China in 2016 ranged between 45 and 46.9 percent.
(B) Correct. In Russia, more than 49 percent of females had a primary
school education in 2016.
(C) Incorrect. The percentage of females with primary school education in
Nigeria in 1986 was less than 45 percent.
(D) Incorrect. The percentage of females with primary school education in
Iran in 1986 was less than 45 percent.

80 AP Comparative Government & Politics Practice Exam


Question 49
Skill Learning Objective Topic
3.C: Explain patterns and trends LEG-3.B: Explain how Adaptation of
in data to draw conclusions. governments adapt social Social Policies
policies to address political,
cultural, and economic
changes.
(A) Incorrect. The Iranian government did not stop females from attending
primary or secondary school. The Iranian government created
education policies that encouraged females to attend primary school,
secondary school, and university. The map indicates an increase in the
percentage of females attending primary school, not a decrease.
(B) Incorrect. Hardliners in the 1990s did not refuse to allow females the
right to attend school or sporting events. The Iranian government
created education policies that encouraged females to attend primary
school, secondary school, and university. The map indicates an increase
in the percentage of females attending primary school, not a decrease.
(C) Incorrect. The supreme leader did not change national laws to allow
women the right to vote and attend school. The Iranian government
created education policies that encouraged females to attend primary
school, secondary school, and university.
(D) Correct. The Iranian government created education policies that
encouraged females to attend primary school, secondary school, and
university.

AP Comparative Government & Politics Practice Exam 81


Question 50
Skill Learning Objective Topic
2.C: Explain the implications of PAU-4.B: Explain how Role of Political
the similarities and differences political party systems and Party Systems
between countries with different memberships link citizen
political systems, principles, participation to policy-
institutions, processes, policies, making.
and behaviors.
(A) Incorrect. In Russia, electoral rules have made it difficult for political
parties to effectively compete pushing the system toward a single party
dominant system, not a multiparty system. In the United Kingdom,
electoral rules allow regional parties to win legislative seats, but not a
majority of seats.
(B) Incorrect. It is true that electoral rules such as a relatively high
threshold in Russia have made it difficult for small parties to compete
or gain representation in the legislature. However, in the United
Kingdom, single member districts have not resulted in a legislature
with only two parties. The United Kingdom has a party system with
many parties, but two main parties repeatedly win the most legislative
seats.
(C) Incorrect. In Russia, electoral rules have resulted in a one-party
dominant system rather than a two-party dominant system. In the
United Kingdom, electoral rules allow regional and small parties to win
legislative seats while maintaining a system with two major parties.
(D) Correct. In Russia, electoral rules make it difficult for parties to
effectively compete, limiting the spectrum of representation in the
national legislature, while in the United Kingdom, electoral rules allow
regional and small parties to win legislative seats while maintaining a
system with two major parties.

82 AP Comparative Government & Politics Practice Exam


Question 51
Skill Learning Objective Topic
1.C: Compare political systems, PAU-3.B: Compare Comparing
principles, institutions, processes, institutional relations Parliamentary,
policies, and behaviors. among parliamentary, Presidential,
presidential, and semi- and Semi-
presidential systems. Presidential
Systems
(A) Incorrect. In a parliamentary system there may be imposed time
deadlines on calling new elections, whereas in a presidential system,
elections have a set schedule. In both a presidential and parliamentary
system, the legislative branch has the sole power to pass or reject
legislation.
(B) Incorrect. In a presidential system the lower house of the legislature may
pass an impeachment resolution, whereas in a parliamentary system
a party may call for a vote of confidence. In both a presidential and
parliamentary system, the legislative branch has the sole power to pass
or reject legislation.
(C) Correct. In both a presidential and parliamentary system, the legislative
branch has the sole power to pass or reject legislation.
(D) Incorrect. The Supreme Court may not remove the executive from office
in either a presidential or parliamentary system. In both a presidential
and parliamentary system, the legislative branch has the sole power to
pass or reject legislation.

AP Comparative Government & Politics Practice Exam 83


Question 52
Skill Learning Objective Topic
1.D: Describe political systems, LEG-4.A: Explain political Causes and
principles, institutions, processes, causes and consequences of Effects of
policies, and behaviors of a demographic changes. Demographic
course country. Change
(A) Incorrect. The United Kingdom’s most significant losses of jobs in
the natural-resource sector happened decades ago. An effect of a
demographic change in the United Kingdom has been a positive net
migration of immigrants that has resulted in social and political tensions
in the country.
(B) Incorrect. The British government has not created special economic
zones within the country. An effect of a demographic change in the
United Kingdom has been a positive net migration of immigrants that
has resulted in social and political tensions in the country.
(C) Correct. Positive net migration of immigrants to the United Kingdom
has resulted in social and political tensions in the country.
(D) Incorrect. There has been a demographic population shift from rural to
urban in the United Kingdom, not a shift from urban to rural. Positive
net migration of immigrants to the United Kingdom has resulted in
social and political tensions.

84 AP Comparative Government & Politics Practice Exam


Question 53
Skill Learning Objective Topic
1.E: Explain how political PAU-3.F: Explain how Independent
systems, principles, institutions, legislative powers are Legislatures
processes, policies, and behaviors constrained by other
apply in a course country. institutions and/or
processes, which can affect
legislative independence.
(A) Incorrect. China is unicameral and there is no upper house to constrain
the legislature. The National People’s Congress is recognized by the
Constitution as the highest organ of power but in practice operates
under the control of the Politburo Standing Committee and passes most
legislation without question.
(B) Incorrect. The National People’s Congress of China is elected by
regional legislatures, not directly by the people. Voters only select local
People’s Congresses. The National People’s Congress is recognized by
the Constitution as the highest organ of power but in practice operates
under the control of the Politburo Standing Committee and passes most
legislation without question.
(C) Correct. The National People’s Congress is recognized by the
Constitution as the highest organ of power, but in practice operates
under the control of the Politburo Standing Committee and passes most
legislation without question.
(D) Incorrect. The Chinese Constitution does recognize the National
People’s Congress as the most powerful institution in government.
However, in practice, the National People’s Congress operates under the
control of the Politburo Standing Committee and passes most legislation
without question.

AP Comparative Government & Politics Practice Exam 85


Question 54
Skill Learning Objective Topic
1.E: Explain how political IEF-2.A: Explain how Impact
systems, principles, institutions, social movements and of Social
processes, policies, and behaviors interest groups affect social Movements
apply in a course country. and political change. and Interest
Groups
(A) Incorrect. There were no electoral reforms in the Majles in the 2000s.
The Green Movement in Iran started as a protest against corruption in
the 2009 presidential election.
(B) Incorrect. The Green Movement began in part because citizens believed
the presidential election was fraudulent. President Ahmadinejad won
reelection and was not removed from office. The Green Movement
in Iran started as a protest against corruption in the 2009 presidential
election.
(C) Incorrect. The Guardian Council vets candidates regularly. While the
vetting process is sometimes seen as controversial, it was not the cause
of the Green Movement. The Green Movement in Iran started as a
protest against corruption in the 2009 presidential election.
(D) Correct. The Green Movement in Iran started as a protest against
corruption in the 2009 presidential election.

Question 55
Skill Learning Objective Topic
1.D: Describe political systems, LEG-2.A: Describe Political
principles, institutions, processes, politically relevant social and Social
policies, and behaviors of a cleavages. Cleavages
course country.
(A) Incorrect. The Shi’a population makes up most of the population in
both the North and South. Despite official recognition of Judaism,
Christianity, and Zoroastrianism, a religious division exists within the
Shi’a Muslim majority.
(B) Incorrect. The Azerbaijanis and Kurds are small, ethnic minority
populations in Iran and do not threaten the stability of the
regime. Despite official recognition of Judaism, Christianity, and
Zoroastrianism, a religious division exists within the Shi’a Muslim
majority.
(C) Correct. Despite official recognition of Judaism, Christianity, and
Zoroastrianism, a religious division exists within the Shi’a Muslim
majority.
(D) Incorrect. Most Iranians are Persian, which does not create a significant
cleavage with the smaller Arab population. Despite official recognition
of Judaism, Christianity, and Zoroastrianism, a religious division exists
within the Shi’a Muslim majority.

86 AP Comparative Government & Politics Practice Exam


Answer Key and Question Alignment to Course Framework
Item Sequence Answer Skill Learning Topic
Key Text Objective
1 C 1.B PAU-2.A Federal and Unitary Systems
2 A 1.A MPA-1.A The Practice of Political Scientists
3 C 1.C IEF-2.B Pluralist and Corporatist Interests
4 D 1.D DEM-2.B Objectives of Election Rules
5 C 1.B LEG-5.A Impact of Natural Resources
6 A 2.A DEM-2.B Objectives of Election Rules
7 B 2.B DEM-2.A Electoral Systems and Rules
8 A 1.E LEG-4.A Causes and Effects of Demographic
Change
9 D 1.E DEM-1.C Civil Rights and Civil Liberties
10 C 2.A IEF-3.B Political Responses to Global
Market Forces
11 C 1.D PAU-3.G Judicial Systems
12 B 1.A IEF-2.B Pluralist and Corporatist Interests
13 D 3.B MPA-1.A The Practice of Political Scientists
14 B 3.D LEG-1.B Sustaining Legitimacy
15 C 3.E MPA-1.A The Practice of Political Scientists
16 B 2.A PAU-3.F Independent Legislatures
17 D 1.C PAU-3.A Parliamentary, Presidential, and
Semi-Presidential Systems
18 C 1.B PAU-3.C Executive Term Limits
19 C 2.C LEG-2.B Political and Social Cleavages
20 D 2.A LEG-1.A Political Legitimacy
21 A 1.E IEF-3.E Policies and Economic
Liberalization
22 B 2.A PAU-4.B Role of Political Party Systems
23 B 4.A IEF-2.A Executive Systems
24 C 4.B PAU-3.C Executive Systems
25 A 4.C LEG-1.B Sustaining Legitimacy
26 D 1.B PAU-1.C Democratization
27 D 1.C LEG-3.A International and Supranational
Organizations
28 B 1.C IEF-1.C Political Ideologies
29 D 1.E DEM-2.B Objectives of Election Rules
30 C 1.B LEG-3.A International and Supranational
Organizations
31 D 2.C PAU-3.G Judicial Systems
32 C 2.C PAU-4.B Objectives of Election Rules
33 C 2.A DEM-1.C Civil Rights and Civil Liberties
34 C 1.B IEF-3.C Challenges from Globalization

AP Comparative Government & Politics Practice Exam 87


Item Sequence Answer Skill Learning Topic
Key Text Objective
35 D 3.A MPA-1.A The Practice of Political Scientists
36 D 3.D PAU-3.G Judicial Systems
37 C 3.D LEG-4.A Causes and Effects of Demographic
Change
38 A 1.A DEM-1.C Civil Rights and Civil Liberties
39 C 2.A PAU-3.D Removal of Executives
40 D 2.B PAU-3.E Legislative Systems
41 B 2.A PAU-3.A Parliamentary, Presidential, and
Semi-Presidential Systems
42 D 4.A MPA-1.A The Practice of Political Scientists
43 B 4.C IEF-3.C Challenges from Globalization
44 D 1.A MPA-1.A The Practice of Political Scientists
45 A 2.A PAU-2.A Federal and Unitary Systems
46 C 2.A IEF-1.B Civil Society
47 D 1.E DEM-1.B Forces that Impact Political
Participation
48 B 3.B MPA-1.A The Practice of Political Scientists
49 D 3.C LEG-3.B Adaptation of Social Policies
50 D 2.C PAU-4.B Role of Political Party Systems
51 C 1.C PAU-3.B Comparing Parliamentary,
Presidential, and Semi-Presidential
Systems
52 C 1.D LEG-4.A Causes and Effects of Demographic
Change
53 C 1.E PAU-3.F Independent Legislatures
54 D 1.E IEF-2.A Impact of Social Movements and
Interest Groups
55 C 1.D LEG-2.A Political and Social Cleavages

88 AP Comparative Government & Politics Practice Exam


Free-Response Section
Scoring Guidelines

Question 1: Conceptual Analysis 4 points


Learning Objectives: MPA-1.A IEF-3.E LEG-1.B

(A) Describe the Gini Index. 1 point


1.A
Acceptable descriptions include:
• A measure of the distribution of income or wealth in a country, most
commonly used to measure inequality.
(B) Describe a difference between the Gini Index and the Human Development 1 point
Index (HDI). 1.C

Acceptable descriptions include:


• The Gini Index is used to measure income inequality and the HDI takes into
consideration many factors, including life expectancy, knowledge or standard
of living.
• The Gini Index is used to measure income inequality, and the HDI measures
dimensions of a country’s social and economic achievement.
Note: A response does not have to include all factors of HDI to earn credit.
(C) Explain how a government policy could affect a state’s Gini Index score. 1 point
1.B
Acceptable explanations include:
• Governmental tax policies can favor certain groups, such as wealthier
individuals.
• Governmental tax policies can favor certain groups, such as poorer
individuals.
• Governments can create social welfare programs aimed at limiting disparity.
• Governments can provide incentives for wealthy individuals and corporations
to make great profit, thereby exacerbating inequality.
• Governments decrease public spending or adopt austerity measures that
limit support for the lower classes, increasing income inequality.
• Governments may adopt socialist or communist policies that decrease
economic inequality.
• Governments may take control over all aspects of the economy in order to
eliminate economic inequality.

AP Comparative Government & Politics Practice Exam 89


(D) Explain how changes in the Gini score reflect state politics and could enhance 1 point
or diminish an authoritarian regime’s stability. 1.B

Acceptable explanations include:


• Authoritarian regimes can maintain legitimacy if there is equality, so changing
a score toward greater equality could result in greater regime stability.
• If inequality becomes greater in an authoritarian regime, then citizens might
protest and not recognize the government and even overthrow the regime.
Total for question 1 4 points

90 AP Comparative Government & Politics Practice Exam


Question 2: Quantitative Analysis 5 points
Learning Objectives: MPA-1.A DEM-1.A LEG-1.B

(A) Using the data in the graph, identify the year with the lower turnout. 1 point
3.A
An identification includes:
• 1993
(B) Using the data in the graph, describe the relationship between total vote and 1 point
voting-age population in Mexico between 1993 and 2017. 3.B

Acceptable descriptions include:


• The voting-age population was always higher than the total vote in Mexico
between 1993 and 2017.
• The voting-age population and the total vote steadily increase between 1993
and 2017.
• There is a positive correlation between total vote and voting-age population.
(C) Describe political participation. 1 point
1.A
Acceptable descriptions include:
• Political participation includes a wide range of political activities through
which people express their views.
• Political participation can be voluntary or coerced.
• Political participation may occur at the individual or group level.
• Political participation includes but is not limited to voting, protests, or acts of
political violence.
(D) Using the data in the graph, draw a conclusion about political participation in 1 point
legislative elections in Mexico. 3.C

Acceptable descriptions include:


• Low levels of participation in legislative elections indicate a low level of
political efficacy in Mexico.
• Low levels of participation in legislative elections indicate that people in
Mexico do not view the legislature as an important political institution in
Mexico.
• Political participation in legislative elections in Mexico is low because of
citizens’ overall low efficacy and belief their votes do not count after decades
of corruption and vote fraud under the PRI.
• Mexico’s SMD plurality voting system discourages high voter turnout as it
fosters the creation of two large catchall parties.

AP Comparative Government & Politics Practice Exam 91


(E) Explain what the data imply about why democracies encourage political 1 point
participation. 3.D

Acceptable explanations include:


• Democracies encourage political participation so that many different citizen
views are heard in a variety of ways in order to promote legitimacy in the
regime.
• Democracies encourage participation because a democracy is based on the
people, and their participation is necessary to maintain legitimacy in the
regime.
• Democracies encourage participation because if engagement is low, the
legitimacy of the regime will weaken.
Total for question 2 5 points

92 AP Comparative Government & Politics Practice Exam


Question 3: Comparative Analysis 5 points
Learning Objectives: IEF-3.A LEG-3.A

(A) Define economic globalization. 1 point


1.B
Acceptable definitions include:
• Economic networks that are growing more interconnected
• A worldwide market with actors unconstrained by political borders
• A reduction in state control over economies
(B) Explain how state membership in an international economic organization has 2 points
influenced economic policies in two different AP Comparative Government 1.E

and Politics course countries.

Acceptable explanations include the following (max one point per country):
• State membership in the International Monetary Fund (IMF) for Iran/Mexico/
Nigeria/Russia/the United Kingdom has promoted economic liberalization
policies.
• State membership in the World Bank for Iran/Mexico/Nigeria/Russia/the
United Kingdom has promoted economic liberalization policies.
• State membership in the World Trade Organization (WTO) for China/Mexico/
Nigeria/Russia/the United Kingdom has promoted economic liberalization
policies.
• China has changed economic policies to comply with the WTO in order to be
admitted to the organization.
• Nigeria has changed economic policies to comply with the WTO in order to
be admitted to the organization.
• Nigeria has changed economic policies to comply with the World Bank in
order to receive financial assistance.
• In order to accept emergency loans from the IMF, Russia’s government had
to adopt structural adjustment programs that reduced tariffs on imports and
reduced government subsidies to domestic businesses.
• In order to accept emergency loans from the IMF, Mexico’s government had
to adopt structural adjustment programs that reduced tariffs on imports and
reduced government subsidies to domestic businesses.
• The UK had to alter regulatory and economic policies to be in line with those
of the EU.
• The UK adopted more neoliberal economic policies as a member of the EU.

AP Comparative Government & Politics Practice Exam 93


(C) Explain why each of the two AP Comparative Government and Politics course 2 points
countries described in (B) would choose state membership in an international 2.A

economic organization. 2.B

2.C
Acceptable explanations include the following (max one point per country):
• After joining the WTO, the size of China’s economy grew immensely.
• After joining the WTO, China’s per capita GDP grew substantially because of
its ability to export new products to global consumers.
• A condition for Mexico’s receiving IMF emergency loans was to reduce
government subsidies to domestic industries and to reduce tariffs on
imports. This led to the government’s support of the North American Free
Trade Agreement that caused Mexico’s government to further embrace
neoliberal economic reforms as its per capita GDP grew.
• The UK joined the EU in order to gain easier access to European markets. The
EU became the UK’s primary trading partner, and London established itself as
the financial hub of Europe.
• Russia joined the WTO to increase access to global trade and be accepted
into the international community.
Total for question 3 5 points

94 AP Comparative Government & Politics Practice Exam


Question 4: Argument Essay 5 points
Learning Objectives: PAU-1.A DEM-2.B

Reporting
Scoring Criteria
Category
Row A 0 points 1 point
Claim/Thesis Does not meet the criteria for one point. Responds to the prompt with a defensible claim or thesis that establishes a line
(0–1 points) of reasoning.
Decision Rules and Scoring Notes
5.A
Responses that do not earn this point: Responses that earn this point:
• Only restate the prompt. • Responds to the prompt rather than restating or rephrasing the prompt and
• Do not make a claim that responds to the prompt. establishes a line of reasoning.
• Provide a defensible claim or thesis that establishes a line of reasoning about
whether a country should or should not join a supranational organization,
using one or more of the provided course concepts: sovereignty;
globalization; or economic liberalization.
Examples that do not earn this point: Examples that earn this point:
Restate the prompt • “Two-party systems are better for the democratization process than
• “Multiparty systems are better for democracies than two-party systems.” multiparty systems because they provide much needed stability during a time
• “Two-party systems are helpful in the democratization process.” of transition.”
• “Two-party systems are better for the democratization process than
Do not respond to the prompt
multiparty systems because they provide clear alternatives to government.”
• “Two-party systems and multiparty systems are both found in democratic
regimes.” • “Multiparty systems are better for the democratization process because they
allow real electoral competition.”
• “Multiparty systems are better for the democratization process because they
allow for leaders to be held accountable through competitive elections.”
• “Multiparty systems enhance the democratic process because they provide
a range of options and allow many views to be represented strengthening
legitimacy.”
Additional Notes:
• The claim or thesis must consist of one or more sentences that can be located anywhere in the response.
• The claim or thesis that meets the criteria can be awarded the point whether or not the rest of the response successfully conveys that line of reasoning.

AP Comparative Government & Politics Practice Exam


95
Reporting
Scoring Criteria
Category
Row B 0 points 1 point 2 points
Evidence Does not meet the criteria for one point. Provides one piece of specific and relevant evidence Provides two pieces of specific and relevant evidence
(0–2 points) from a course country relevant to one of the course from one or more course countries relevant to one or
concepts in the prompt. more of the course concepts in the prompt.
Decision Rules and Scoring Notes
Responses that do not earn points: Responses that earn 1 or 2 points:
• Do not provide any accurate evidence. • Provide specific and relevant evidence from required course countries, relevant to the course concepts
• Provide evidence that is not relevant to the course in the prompt.
concepts in the prompt.

96 AP Comparative Government & Politics Practice Exam


Examples that do not earn points: Examples of acceptable specific and relevant evidence (one example is one piece of evidence):
Not specific • “One of the first major events of Mexico’s transition to democracy was the development of a multiparty system.”
• “Mexico has transitioned to a democracy.” • “The development of a multiparty system in Mexico in the late 1990s and early 2000s helped the country
complete the democratization process.”
Provide evidence that is not relevant to course
concepts in the prompt • “After the PRI lost in Mexico, the democratization process was able to move forward.”
• “The United Kingdom is a postmaterialist country • “Mexico’s transition to a multiparty system was important to its democratization process.”
with different types of development issues.” • “In Nigeria after the 1999 constitution, a multiparty system developed, and citizens were able to elect leaders in
competitive elections.
• “In China, the regime promotes economic
growth.” • “In Nigeria after the 1999 constitution, a multiparty system developed.”
• “The 2015 presidential elections in Nigeria were important in the democratic transition process.

Additional Notes:
• A response does not need to earn the point in Row A to earn points in Row B.
• A response does not need to explain the relationship between the evidence and the claim or thesis to earn points in Row B. (That explanation is evaluated in Row C.)
Reporting
Scoring Criteria
Category
Row C 0 points 1 point
Reasoning Does not meet the criteria for one point. Explains how or why the evidence supports the claim or thesis.
(0–1 points)
Decision Rules and Scoring Notes
5.C
Responses that do not earn this point: Responses that earn this point:
• Include evidence but offer no reasoning to connect the evidence to the claim • Explain the relationship between the evidence provided and the claim or
or thesis. thesis.
• Restate the prompt without explaining how the evidence supports the claim
or thesis.
Examples of reasoning that explain how evidence supports the claim or
thesis:
• “As Mexico transitioned to democracy and a multiparty system developed,
citizens were finally able to hold leaders accountable.
• “As Mexico transitioned to democracy and a multiparty system developed, state
legitimacy was developed because election results were not predetermined.”
• “In Mexico under the PRI, the democratization process was able to develop
because the PRI allowed for a stable environment in which economic development
could happen.”
• “In Nigeria after the 1999 constitution, a multiparty system developed, and
citizens were able to elect leaders in competitive elections. This facilitated the
democratization process because it provided legitimacy to the government.”
• “The peaceful transfer of power in Nigeria after the 2015 presidential election
was a major development in the democratic transition because it was the first
peaceful transfer of power from one party to another.”
Additional Notes:
• To earn this point, the response must have a defensible claim or thesis (earned the point in Row A).
• The explanation of the relationship between one piece of evidence and the claim or thesis is sufficient to earn this point.

AP Comparative Government & Politics Practice Exam


97
Reporting
Scoring Criteria
Category
Row D 0 points 1 point
Responds Does not meet the criteria for one point. Responds to an opposing or alternate perspective using refutation, concession,
to Alternate or rebuttal.
Perspectives Decision Rules and Scoring Notes
(0–1 points)
Responses that do not earn this point: Responses that earn this point:
5.D • Restate the opposite of the claim or thesis. • Must describe an alternate perspective AND refute, concede, or rebut that
• May identify or describe an alternate perspective but do not refute, concede, perspective.
or rebut that perspective.

98 AP Comparative Government & Politics Practice Exam


Examples of responses that do not earn the point: Examples of acceptable responses to an alternate perspective may include:
Restate the opposite of the claim or thesis • “Although two-party systems may promote stability that is necessary for
• “Even with evidence to the contrary, many try to argue that countries should economic growth, the democratization process is best promoted through
not join supranational organizations.” multiparty systems that offer citizens and minority views a real choice.”
Describe an alternate perspective but does not refute, concede, or rebut that • “Some argue that multiparty systems are best for the democratization
perspective process because they provide a range of party options for voters, but two-
• “There are those who argue that countries should not join supranational party systems are better because they are more suited to promoting stable
organizations and that joining would be a negative thing for that country. governments and have only one party in power at a time with no coalitions.”

Additional Notes:
• To earn this point, the response must have a defensible claim or thesis (earned the point in Row A).
• Responses that demonstrate an incorrect understanding of the alternate perspective do not earn this point.
Contact Us
apcentral.collegeboard.org

New York Office


250 Vesey Street
New York, NY 10281
212-713-8000
212-713-8277/55 (Fax)

AP Services for Educators


P.O. Box 6671
Princeton, NJ 08541-6671
877-274-6474 (toll free in the United States and Canada)
212-632-1781
610-290-8979 (fax)
Email: apexams@info.collegeboard.org

Call Center Hours


M–F, 8 a.m. to 8 p.m. ET
April 20 to May 29, 2020, M–F, 7 a.m. to 9 p.m. ET

AP Canada Office
2950 Douglas Street, Suite 550
Victoria, BC, Canada V8T 4N4
250-472-8561
800-667-4548 (toll free in Canada only)
Email: gewonus@ap.ca

College Board International


Serving all countries outside the United States and Canada
250 Vesey Street
New York, NY 10281
212-373-8738
Email: international@collegeboard.org

AP Comparative Government & Politics Practice Exam 99


apcentral.collegeboard.org

You might also like